На habr ранее активно обсуждалась теория Вселенной, осциллирующей в черной дыре, которая развивается в ряде работ, в том числе моих с соавторами (но не только). Чего стоит дискуссия от 2018 года на 600 комментариев «Жизнь внутри черной дыры» (отмечу, что я не инициировал этот пост, просто меня спросили — не возражаю ли я, а я, конечно, вовсе нет). Там, конечно, много странных заявлений, но я не принимал прямое участие в этой дискуссии, потому что был слишком занят дальнейшим развитием теории. Но в этом году ситуация изменилась: работа над моделью циклической Вселенной с переменной гравитационной массой для меня практически завершена. Она подробно изложена в книге «Осциллирующая Вселенная», которая опубликована в бумажном и электронном варианте издательством Челябинского государственного университета в феврале 2023 года. Книгу (со свежими уточнениями на 25 сентября 2023 года) можно скачать на сайте Пущинской обсерватории.

Так как сейчас готовится издание книги на английском языке, то я решил обсудить книгу на таких площадках, как Астрофорум и DxDy. Благодаря этим полезным обсуждениям, в книгу были внесены существенные исправления и уточнения, и даже написан новый важный параграф 20.3, где было показано, что наибольшей энергией во Вселенной обладают килогерцовые гравитационные волны, образующие главный пик реликтового фона. Не скрою, что такие обсуждения отнимают немало сил и нервов, потому что на одного умного собеседника, которые задает хорошие вопросы, находится с десяток буквальных гениев, которые снисходительно объясняют мне, что я не прав и забыл таблицу умножения. Тем не менее, вспомнив бурное обсуждение на Хабре, я решил, что полезно будет обсудить книгу и здесь. Буду благодарен, если самые храбрые хабровцы посмотрят книгу и выскажутся — тем самым внесут свой вклад в современную космологию. Вы в хорошей компании, потому что на днях я отправил рабочий перевод книги (сделанный с помощью ChatGPT) нобелевскому лауреату Джону Мазеру, и он ответил, что прочитает. Ему я, правда, отправил по почте напечатанную на бумаге (тираж — три экземпляра) демоверсию книги.

В книге «Осциллирующая Вселенная» описывается новейшая космологическая революция, которая началась в 2015 году, когда были открыты гравитационные волны от слияния множества невидимых черных дыр звездных масс, составляющих темную материю космоса. Это открытие легло в основу модели осциллирующей Вселенной, в динамике которой гравитационное излучение и черные дыры играют ключевую роль. Суть космологической революции XXI века — смена модели одноразовой непрочной Вселенной, тлеющие обломки которой разлетаются по бесконечной холодной пустыне, на циклическую космологию, которая описывает вечную Вселенную, пульсирующую внутри огромной черной дыры. Анализ популяции черных дыр показывает, что циклов у Вселенной было, как минимум, около тысячи, но, вероятно, гораздо больше. Вселенная представляет собой маятник с перетеканием энергии между главными компонентами — негравитирующими гравволнами и гравитирующими черными дырами, что приводит к периодическому изменению общей гравитационной массы нашего мира. Примечательной особенностью обсуждаемой теории является то, что она полностью построена на классической теории гравитации (ОТО) с привлечением хорошо проверенных фактов из квантовой механики и ядерной физики — как в случае эффекта фотодиссоциации атомных ядер. Для создания данной космологической теории не было сделано ни одного предположения вроде существования неизвестных сил, полей или размерностей, а также не было введено ни одной новой фундаментальной константы.

Модель пульсирующей Вселенной решает проблемы механизма Большого взрыва, предотвращения гравитационной сингулярности, природы темной энергии и избегания роста энтропии. Циклическая космология объясняет накопление темной материи в виде черных дыр, а также раннее происхождение сверхмассивных черных дыр. Это позволяет просто вывести галактические соотношения типа Талли‑Фишера. Предсказание циклической космологии о реликтовом фоне низкочастотных гравитационных волн, вызванных массовым слиянием черных дыр при сжатии Вселенной до минимального размера в десять световых лет, уже подтверждено: консорциум NANOGrav недавно обнаружил этот фон наногерцовых волн.

Книга рассчитана на широкого читателя, и ее основной текст почти не содержит формул. Для специалистов в Приложении II размещен обзор по математическим решениям циклической космологии. Этот обзор, а также части III и IV данной книги, базируются на научных работах Н. Горькавого, А. Василькова, Дж. Мазера и С. Тюльбашева:

  1. Gorkavyi, N. “Origin and Acceleration of the Universe without Singularities and Dark Energy”. Bulletin of the American Astronomical Society. 2003, 35, #3.
    http://www.aas.org/publications/baas/v3 ... 02/404.htm

  2. Gorkavyi N., Vasilkov A. “A repulsive force in the Einstein theory”. Monthly Notices of the Royal Astronomical Society. 2016, 461 (3): 2929-2933. https://doi.org/10.1093/mnras/stw1517

  3. Gorkavyi N., Vasilkov A. “A modified Friedmann equation for a system with varying gravitational mass”. Monthly Notices of the Royal Astronomical Society. 2018, 476 (1): 1384-1389. https://doi.org/10.1093/mnras/sty335

  4. Gorkavyi, N., Vasilkov, A., Mather, J. A Possible Solution for the Cosmological Constant Problem. In Proceedings of the 2nd World Summit on Exploring the Dark Side of the Universe (EDSU2018) Eds: B. Vachon and P. Petroff, Point a Pitre, Guadeloupe, France, 25-29 June 2018, https://pos.sissa.it/335/039/pdf , https://doi.org/10.22323/1.335.0039

  5. Горькавый, Н.Н. и Тюльбашев, С.А. «Черные дыры и нейтронные звезды в осциллирующей Вселенной», Астрофизический бюллетень. 2021, т.76, N3, с.285-305.
    https://www.sao.ru/Doc-k8/Science/Publi ... SPB285.pdf
    (Gorkavyi, N.N., Tyul'bashev, S.A. Black holes and neutron stars in an oscillating Universe. Astrophys. Bull., 2021, 76, 229-247. https://doi.org/10.1134/S199034132103007X)

  6. Gorkavyi, N. "Gravitational wave background discovered by NANOGrav as evidence of a cyclic universe". New Astronomy. 2021, 91: 101698. https://doi.org/10.1016/j.newast.2021.101698

  7. Gorkavyi, N. “Accretion of Galaxies around Supermassive Black Holes and a Theoretical Model of the Tully-Fisher and M-Sigma Relations”. Galaxies. 2022, 10: 73.
    https://doi.org/10.3390/galaxies10030073

Результаты, которые легли в основу данной книги, опубликованы в рецензируемых журналах [2,3,5–7], и обсуждались на конференциях, например, Американского астрономического общества [1] и международных симпозиумах, например, «Exploring the Dark Side of the Universe» на французском острове Гваделупа в 2018 году [4], а также на семинарах в ИНАСАН, КрАО и Годдардском центре космических полетов НАСА. Популярному изложению теории циклической Вселенной посвящены две свежие статьи: Николай Горькавый «Вселенная, пульсирующая в черной дыре», «Наука и жизнь», декабрь 2022, с. 2–14; «Цикличность Большого взрыва», «Наука и жизнь», январь 2023, с. 2–11. Хороший обзор статьи [5] и эволюции черных дыр в циклической Вселенной дал на ютуб‑канале QWERTY пулковский астроном Кирилл Масленников: «ЦИКЛИЧЕСКАЯ ВСЕЛЕННАЯ. Что будет после того, как вселенная сожмется?», 18 августа 2021, QWERTY, https://www.youtube.com/watch?v=xWKj5yJRk74 

P. S. Исходя из опыта прошлых интернет обсуждений отмечу:

  1. Да, Вселенная имеет центр, расположенный, видимо, в направлении южного полюса. Рассказывать мне, что Вселенная однородна — не надо, потому что доказана анизотропность постоянной Хаббла с амплитудой в 15%. Плюс «напряжение Хаббла», которое снимает вопрос об однородности Вселенной.

  2. Гравитационные волны ИМЕЮТ энергию, импульс и инертную массу (но есть тонкости интерпретации из‑за псевдотензорности этих величин), но не имеют ГРАВИТАЦИОННУЮ массу (активную, то есть порождающую дополнительное гравполе). Это трактовка не моя, а Эйнштейна. Ссылаться на принцип эквивалентности пассивной инертной и пассивной гравитационной массы не надо — это совсем другие, де факто, ньютоновские массы, которые ОТО упразднила, введя искривленное пространство и движение тел по геодезической безотносительно от их масс.

  3. Ссылаться на малограмотные критические польские интернет‑статьи Абрамовича и К тоже малополезно. Заявления, что я использовал якобы нековариантное определение ускорения, разбиваются о тот факт, что выражение, предложенное Абрамовичем, в пределе слабых полей (в этом приближении и сделана наша статья 2016 года) совпадает с использованной нами формулой. Ну, просто малограмотный человек.

P.S. 30 ноября появилась моя статья (пока в интернете), специально посвященная энтропии Вселенной и черных дыр: "Entropy of Black Holes and an Oscillating Universe"

The model of an oscillating universe proposed by A. Friedmann in 1922 dominated cosmology for a long time. R. Tolman put forward a thermodynamic argument against the cyclic cosmological model, believing that entropy should accumulate from cycle to cycle. J. Bekenstein and S. Hawking discovered a tremendous amount of entropy associated with black holes. This discovery led to the conclusion that the majority of the Universe's entropy is contained in black holes. N. Poplawski considers a Universe that oscillates within a black hole. This paper analyzes the relationship between the entropy of black holes and the oscillating Universe inside a large black hole. It is demonstrated that the entropy of a black hole can only be defined for an external observer. For an observer inside the black hole, its entropy is zero. This opens up the possibility of constructing a cyclic cosmology in which entropy and the size of the visible part of the Universe change strictly periodically. This does not violate the second law of thermodynamics because the continuous accumulation of entropy from cycle to cycle pertains to the surface of the big black hole, which is invisible to the internal observer.

https://www.preprints.org/manuscript/202311.1978/v1

Комментарии (232)


  1. eugenk
    28.11.2023 17:50
    +7

    Николай, Вы теперь и на хабре ??? Страшно рад ! А то тут бываю всё-таки существенно чаще чем в ЖЖ.


  1. AlexeyK77
    28.11.2023 17:50
    +7

    просто заметка: в космологии индуизма\буддизма вселенная имеет циклическую природу от изначального расширения до коллапса в конце.


    1. sim2q
      28.11.2023 17:50
      +1

      в буддизме там ещё и бесконечность вселенных и какие то страшного порядка вложенности


      1. Ubudragon
        28.11.2023 17:50
        +1

        ничего, скоро и это математически опишут


        1. sim2q
          28.11.2023 17:50

          это же уже следствие процесса - пусть описывают :)


  1. Chelidonium
    28.11.2023 17:50
    +1

    Как здорово что открываются новые обстоятельства


  1. MarinaToshina
    28.11.2023 17:50
    +1

    Так а что ж, разумная жизнь и сверхцивилизации, как минимум 1000 раз появлялись и исчезали?


    1. Gorkavyi Автор
      28.11.2023 17:50
      +4

      Академик Сахаров, который занимался и космологией, был сторонником циклической Вселенной, и он первый, насколько мне известно, среди ученых поднял вопрос о выживании цивилизации (или информации о ней) при переходе к следующему циклу. Это есть в его собрании сочинений (и конечно, цитаты оттуда в моей книжке). Но в те времена детали космологического цикла были неизвестны. В нашей циклической модели открывается возможность сохранения цивилизаций на периферии Вселенной - "в сфере богов", если цивилизация имеет двигатель, который затормозит ее падение при сжатии Вселенной. Это практически не исследовано, но такая возможность, я полагаю, есть. Так что не исключено, то во Вселенной существуют цивилизации, переходящие из цикла в цикл.


      1. RainCarNation
        28.11.2023 17:50
        +4

        А как это выглядит можно почитать у Пола Андерсона в Тау-Ноль)


        1. dragonnur
          28.11.2023 17:50

          Или у, емнимс, ВанВогта.


        1. saboteur_kiev
          28.11.2023 17:50

          или Заповедник Гоблинов.


        1. Vold2D
          28.11.2023 17:50

          Там у них случайно получилось. То есть это была бага, а не фича.


        1. Spaceoddity
          28.11.2023 17:50

          Или у Лю Цысиня

          Там много из этой области: гравитационная мембрана, карманные вселенные, перманентно возрождающаяся трисолярианская цивилизация))


          1. MishaRash
            28.11.2023 17:50

            У него там хватает и ляпов — чего стоит один проект снижения скорости света (предельной) меньше орбитальной скорости Земли. В "Тёмном лесе" было как-то наиболее интересно и наименее сомнительно (хотя значительная часть развития сюжета казалась медленной и предсказуемой).


            1. Spaceoddity
              28.11.2023 17:50

              Ну извините, ляпов хватает и в "Марсианине". По крайней мере трилогия заставляет задуматься - в наше время это уже немало))


              1. MishaRash
                28.11.2023 17:50

                В "Марсианине" мне было не так заметно (возможно, из-за специализации). Что там наименее корректное, по-вашему?


                1. Astroscope
                  28.11.2023 17:50

                  Что там наименее корректное, по-вашему?

                  Вопрос не ко мне, но вставлю свои пять копеек: самое начало с бурей. Атмосфера на сегодня кажется слабоватой для такого. Но я бы не придирался - настолько твердой фантастики еще поискать, так что если ради художественности и читабельности и пришлось пойти на некоторые уступки, то масштаб этих уступок все равно крайне невелик, а читабельность крайне высока.


                  1. Spaceoddity
                    28.11.2023 17:50

                    Характерная особенность атмосферы Марса — постоянное присутствие пыли; согласно спектральным измерениям, размер пылевых частиц оценивается в 1,5 мкм. Малая сила тяжести позволяет даже разреженным потокам воздуха поднимать огромные облака пыли на высоту до 50 км. А ветры, являющиеся одним из проявлений перепада температур, часто дуют над поверхностью планеты (особенно в конце весны — начале лета в южном полушарии, когда разница температур между полушариями особенно резкая), и их скорость доходит до 100 м/с.


                    1. ksbes
                      28.11.2023 17:50

                      Ветер при 100 м/с, но при плотности 0,1 атм - это очень грубо как ветер в 10 м/с при давлении 1 атм. Что явно недостаточно, чтоб создать угрозу опрокидывания ракеты.


                      1. Spaceoddity
                        28.11.2023 17:50

                        А по-моему вполне достаточно. Тут много нюансов... Ветер может дуть и порывами и эти порывы вполне могут войти в резонанс с колебаниями корпуса ракеты.

                        https://ru.wikipedia.org/wiki/Такомский_мост

                        Ну и давление там 1/170 (а не 1/10). Но вы, по-моему, не учитываете массу пыли, поднятой в атмосферу этими ветрами. А пыль тоже подчиняется закону сохранения импульса...


                      1. PrinceKorwin
                        28.11.2023 17:50

                        Чтобы порывы вызвали резонанс они должны быть не только сильными, но и частотными. Но порывы на то и порывы, что они неожиданные и не частотные.


                      1. Spaceoddity
                        28.11.2023 17:50

                        А почему порывы не частотные? Только из-за того что вы не можете корректно просчитать движения атмосферных масс и поэтому они вам кажутся случайными?

                        Впрочем, турбулентность - один из сложнейших разделов современной физики. Тем более в условиях другой планеты.

                        Но ссылочку на один инцидент я оставил чуть выше. Порывы может и неожиданные и не частотные, но земные мосты они могут вполне успешно разрушать...


                1. Spaceoddity
                  28.11.2023 17:50

                  Наименее корректное - продырявливание перчатки скафандра и "орбитальное маневрирование")) Удержать вектор тяги в таких условиях - ну конкретной "голивудщиной" пахнуло...

                  На самом деле, мои основные претензии к роману - его вторичность. И это даже если забить на аллюзии с Даниэлем Дефо. Мне всё это сильно напомнило "Лунную пыль" Кларка.

                  Вообще, имхо, главное заблуждение о космосе (как среди аудитории, так и среди "создателей контента") - это "ледяной холод" в открытом космосе)) Хотя бы с этим "Марсианин" справился (главный герой всё-таки мгновенно не окоченел, когда дырку в скафандре проделал).


                  1. ksbes
                    28.11.2023 17:50

                    Ну это фильм - а как оно в книге интересно? Я не читал. Может там пообоснованее всё.


                    1. CaptainFlint
                      28.11.2023 17:50

                      В книге этого вообще не было.

                      Спойлер

                      Он там лишь предложил эту идею, но в конечном итоге его заставили сидеть внутри, пристёгнутым к креслу, пока один из участников влетел прямо внутрь аппарата, прицепился, и они вместе двинулись обратно.

                      Вообще, книга намного более технически корректная, чем фильм, плюс там приходилось решать гораздо больше проблем, и сами эти проблемы были сложнее и требовали куда больше усилий и смекалки. Зачастую до уровня "да охренеть, неужели и из этого он умудрится выкрутиться?!" Фильм очень сильно всё упростил и уголливудил. С другой стороны, книга мне показалась несколько скучнее; там заметно меньше "оживляжа" на единицу времени, чем в фильме. Но не настолько, чтобы хотелось забросить, читал с интересом.


                      1. Zenitchik
                        28.11.2023 17:50

                        Я скажу по другому: почти ничто из того, за что мы любили книгу, не вошло в фильм.


      1. Earthsea
        28.11.2023 17:50
        +4

        он первый, насколько мне известно, среди ученых поднял вопрос о выживании цивилизации (или информации о ней) при переходе к следующему циклу

        До такого перехода еще дожить нужно. Не факт, что информация о цивилизации, и тем более сама цивилизация, доживет хотя бы до смерти звезды.


  1. evgeniytregubenko
    28.11.2023 17:50
    +9

    доказана анизотропность постоянной Хаббла с амплитудой в 15%

    Подскажите пожалуйста, где об этом можно почитать.


    1. Gorkavyi Автор
      28.11.2023 17:50
      +6

      В моей книге, конечно, глава 16 и 17, а там даны ссылки на оригинальные работы: Мигкаса и др и т.п.


  1. MorgooN7
    28.11.2023 17:50

    Интересно, спасибо!


  1. kryvichh
    28.11.2023 17:50
    +1

    Подождём ещё несколько лет, и ChatGPT 10.0 даст ответ и на этот вопрос.


    1. mirrr
      28.11.2023 17:50
      +37

      42


      1. novoselov
        28.11.2023 17:50
        +1

        В некоторых индийских трактатах ответ 84 (цикла включая текущий), так что видимо Deep Thought был запущен не в этом цикле, а значительно раньше :)


    1. SFelix
      28.11.2023 17:50
      +1

      нет


    1. AlexanderS
      28.11.2023 17:50
      +6

      А не получится как в рассказе Айзека Азимова "Последний вопрос"?


      1. skywalk7
        28.11.2023 17:50
        +1

        Тут, скорее, получится как в рассказе "Собысчас" у Лема :D


        1. Spaceoddity
          28.11.2023 17:50
          +1

          Артур Кларк "Девять миллиардов имён Бога"

          Роберт Шекли "Верный вопрос"

          тема в НФ довольно избитая))


  1. Ilusha
    28.11.2023 17:50
    +1

    Подсел на подкасты с Сурдиным. Был бы счастлив послушать о вашей теории.


    1. Gorkavyi Автор
      28.11.2023 17:50
      +9

      Сам не подкастирую, это требует времени. Советую послушать пулковского астронома Кирилла Масленникова: "ЦИКЛИЧЕСКАЯ ВСЕЛЕННАЯ. Что будет после того, как вселенная сожмется?", 18 августа 2021, QWERTY, https://www.youtube.com/watch?v=xWKj5yJRk74


      1. Pshir
        28.11.2023 17:50
        +2

        В недавнем своём видео Кирилл Масленников рассказывал о гигантской сферообразной структуре под названием Хо'олейлана, которая в традиционной модели объясняется проявлением барионных акустических волн. Также он упоминает, что существование подобных структур предсказывается моделью Пенроуза и Гурзадяна. И также упоминал вашу модель, но со словами, что он не знает, предсказывает ли ваша модель что-то подобное. Можете ли вы рассказать подробнее, в чём отличие объяснений существования данной структуры в разных моделях, и делает ли открытие данной структуры какую-либо из моделей более вероятной?


        1. Gorkavyi Автор
          28.11.2023 17:50
          +4

          Я знаю про предсказание огромных колец в теории Пенроуза-Гурзадяна, связанных с массивными реликтовыми дырами, но в детали их расчетов не погружался. В своей теории мы обсуждаем наличие таких огромных реликтовых дыр, но их влияние на фон реликтового излучения не затрагиваем. Если Пенроуз-Гурзадян правы и такие кольца будут надежно подтверждены, то это означает подтверждение всех циклических космологических моделей с массивными реликтовыми дырами. Вряд ли это позволит провести различие между нашей теорией и П-Г, но так как деталей расчетов не знаю, это лишь мое предположение.


          1. Pshir
            28.11.2023 17:50

            Спасибо!


      1. world1709
        28.11.2023 17:50

        В этом видео Кирилл Маслеников, говорит, о том, что вселенная сжимается до размеров куба с гранями 10 световых лет, я правильно понимаю, что вещество там сжато до такой степени, что больше сжиматься не может, почему из этого вещества не образуется черная дыра, ведь оно там сжато до такой степени, что даже разрушаются нейтронные звезды, значит плотность еще выше или не на столько велика, чтобы образовалась черная дыра? Заранее спасибо за ответ.


        1. Gorkavyi Автор
          28.11.2023 17:50

          Из этого вещества размером в десять световых лет давно образовалась черная дыра размером в триллион световых лет - именно там первая космическая скорость равна скорости света. То есть вещество в объеме десятка светолет - это уже глубоко внутри черной дыры и вырваться оттуда можно, только выйдя за пределы обычного решения Шварцшильда - учитывая антигравитацию.


          1. world1709
            28.11.2023 17:50

            Спасибо за ответ!


  1. DGN
    28.11.2023 17:50
    +4

    Мне это напоминает теорию панспермии, мол жизнь зародилась не здесь. Что добавляет сущность и не обьясняет зарождение.

    Итак, у вселенной много циклов. Но некогда всеж был большой взрыв?

    Цивилизации пережившие цикл, очевидно должны были успеть заселить всю вселенную. И где они? (С)


    1. Gorkavyi Автор
      28.11.2023 17:50
      +7

      Большой Взрыв случается всякий раз, когда Вселенная сжимается в очередном цикле. Как возникла эта циклическая система - это уже вопрос другого уровня (одно дело разобраться с устройством часов, а другое дело - понять, как часовщик их собрал). На эту тему можно только высказать смутные гипотезы и в последней главе книги я это делаю.

      Возможно, что есть цивилизации, которые заселили Вселенную или ее части. Но почему вы думаете, что их представители захотят вступить в нами в контакт? Земля разделена, находится в состоянии острых конфликтов, внешнее воздействие только дестабилизирует ситуацию. Они могут ждать, пока поумнеем.


      1. codecity
        28.11.2023 17:50
        +1

        Земля разделена, находится в состоянии острых конфликтов, внешнее воздействие только дестабилизирует ситуацию. Они могут ждать, пока поумнеем.

        А чего ждать, если можно помочь поумнеть? Выступить наставником. Причем жесткое вмешательство не обязательно - достаточно просто предоставить нужную информацию и сообщить - когда вы это все выполните - мы сможем с вами взаимодействовать.

        Люди уже достаточно умны, чтобы это понять.

        Войны имеют вполне объективную причину, как не крути.


        1. Gorkavyi Автор
          28.11.2023 17:50
          +11

          Кому представить ценную информацию, ускоряющую поумнение? Китаю, США, России, Европе, Ирану, Израилю?


          1. codecity
            28.11.2023 17:50
            +1

            Кому представить ценную информацию, ускоряющую поумнение? 

            Всем. Причем не обязательно давать какие-либо данные для повышения боеспособности. Достаточно объявить о своем существовании и поставить нам конкретные цели, которые от нас ожидают. А также предоставить перспективы - зачем нам это нужно.


            1. Gorkavyi Автор
              28.11.2023 17:50
              +1

              Любой полезной информацией в первую очередь воспользуются самые развитые державы, что вызовет дополнительное расслоение. Если никакой информации не давать, а только объявить о существовании развитой цивилизации - и выставить требования, каким нужно удовлетворять для будущего сотрудничества - это с одной стороны, унижение, с другой - вмешательство в чужую цивилизацию с труднопредсказуемыми последствиями. Не исключено, что после ряда неудачных попыток преждевременных контактов, на них был наложен запрет. Это довольно логично. И в любом случае, рассчитывать, что долгоживущая цивилизация непременно распространится везде, трудно - это могут быть замкнутые сообщества в равновесном, стационарном состоянии.


              1. codecity
                28.11.2023 17:50
                +2

                Если никакой информации не давать, а только объявить о существовании развитой цивилизации - и выставить требования

                Это вы себя успокаиваете и пытаетесь убедить что там кто-то есть, пытаетесь найти обоснование почему они могут быть, но не заявлять о своем существовании.

                Не требования - а просто декларировать уважение и причину почему пока не готовы нас включать в большое Вселенское объединение. Что оно есть, но пока не готовы нас принять - и причину над чем поработать, если мы хотим этого союза. И подчеркнуть что все по желанию, можете оставаться изолированными - вас никто не принуждает к развитию.


                1. kipar
                  28.11.2023 17:50

                  что если развитие требует не просто волевого решения? Нет смысла цивилизовывать обезьян или там муравьев.


                  1. codecity
                    28.11.2023 17:50
                    +1

                    Нет смысла цивилизовывать обезьян или там муравьев.

                    Вы принижаете человека. У нас по сути нет явных границ - даже свое собственное устройство мы хотя и не познали до конца, но идем по этому пути и вполне успешно.

                    Другое дело - что самим инопланетянам это может быть не интересно - нафига тратить время, чтобы кого-то там обучать и подтягивать до своего уровня.


                    1. kipar
                      28.11.2023 17:50
                      +3

                      мы не знаем какие там дальше границы. Например им нужно чтобы вся цивилизация объединились в один сверхразум и только тогда с ней есть смысл общаться. Или минимальным порогом является пережить один цикл сжатия вселенной - без этого какие могут быть разговоры и совместные проекты.


                      1. codecity
                        28.11.2023 17:50
                        +4

                        Или минимальным порогом является

                        Или вообще им нафиг не нужно никакое общение на любом уровне. Если они разгадали тайну сознания и могут создавать новые сознания в произвольном количестве - и это не имеет никакой ценности для них - как грязь.


                      1. Keeper10
                        28.11.2023 17:50

                        Артур Кларк, "Конец детства".


                    1. rombell
                      28.11.2023 17:50

                      Очевидно у нас есть границы. Нам для размышлений требуется оперировать какой-то концепцией целиком. Вполне можно сконструировать концепции, просто не помещающиеся в человеческий мозг. Например, математические теоремы, формулировка которых занимает пару томов, невозможно уместить в голове человека. Но вполне уже можно уместить в том же ChatGPT, и, очевидно, в более продвинутых ИИ.


                      1. Zenitchik
                        28.11.2023 17:50

                        Полагаю, не стоит. Не для того теоремы формулировали в виде, удобном для описания классическими алгоритмами, чтобы в "чёрный ящик" их складывать.


                      1. rombell
                        28.11.2023 17:50

                        конечно, не стоит. Речь о том, что границы у мозга вполне есть.


                1. ti_zh_vrach
                  28.11.2023 17:50

                  Почитайте восьмое путешествие Ийона Тихого. Это короткий рассказ.


                1. FruTb
                  28.11.2023 17:50
                  +1

                  Ну даже на земле есть, к примеру, племя Сентинельцев которых так и не цивилизовали.


                1. Anastasiya_lavva
                  28.11.2023 17:50

                  Может, нас уже «пригласили» и декларировали нам свое уважение.. А наши правители возжелали остаться изолированными?.


                  1. Sunrise33g
                    28.11.2023 17:50

                    Я надеюсь, что инопланетные цивилизации будут достаточно проницательны для того, чтобы отделять человека от государства. И чтобы объективно оценить текущую ситуацию на Земле, где люди фактически в рабстве.


                    1. BugM
                      28.11.2023 17:50

                      Вы к ним подходите с позиции человека 21 века. Такой же человек как и вы живший всего несколько тысяч лет назад вашу проблему даже не понял бы. Люди не понимавшие ваших проблем прожили на Земле гораздо дольше чем те кто их мог бы понять. Да и несколько тысяч лет по меркам космоса это смешно.

                      Смотрите глобальнее. Гораздо глобальнее. Чтобы не говорить о муравьях (всем надоело) представьте что у вас на острове живет цивилизация кроманьонцев. Выбраться они с него не могут (океан и все такое), как-то живут. Пока выглядит так что обнести ленточкой и поставить табличку не беспокоить разумнее всего. Наблюдать при этом стоит. Почему нет?


                      1. Sunrise33g
                        28.11.2023 17:50

                        Такой же человек как и вы живший всего несколько тысяч лет назад вашу проблему даже не понял бы.

                        Античные философы это отлично понимали. Скорее всего далеко не только философы, но и много кто ещё. А уже позже запредельно вырос уровень религиозности и запредельно упал уровень осознанности.

                        Чтобы не говорить о муравьях

                        Аналогия с муравьями некорректна по многим причинам.

                        Пока выглядит так что обнести ленточкой и поставить табличку не беспокоить разумнее всего.

                        Это не только не разумно, но и бесчеловечно, лишать разумных существ благ цивилизации по признаку происхождения. Если Вы имеете ввиду ситуацию с сентинельцами, то их не изолируют от мира, снабжают гуманитаркой, и любой сентинелец может получить убежище и гражданство в Индии. Просто местные вошьди их затюкали до невменяемого состояния.

                        И да, я сейчас чувствую себя в похожей ситуации, когда меня пытаются изолировать запереть в выделенной мне резервации. Причём стены возводят с обеих сторон. И это совсем не здорово, скажу я Вам.


                      1. BugM
                        28.11.2023 17:50

                        Античные философы это отлично понимали

                        Вы еще немного назад по времени загляните. Современному человеку как виду сотни тысяч лет. Вы всего пару процентов от этого времени взяли.

                        Это не только не разумно, но и бесчеловечно, лишать разумных существ благ цивилизации по признаку происхождения

                        При этом мы люди как вид именно это и делаем. Причем даже со своим биологическим видом. Я не зря про кроманьонцев написал. Они вроде близкие, но другой вид. И уровень развития как раз несравнимый.

                        Это не только не разумно, но и бесчеловечно, лишать разумных существ благ цивилизации по признаку происхождения

                        Вы опять смотрите сейчас. Снова рекомендую посмотреть хотя бы на несколько тысяч лет. Лучше на несколько десятков тысяч.

                        Любая цивилизация развитая хотя бы до путешествий в пределах галактики должна оперировать такими временными промежутками легко. Просто из-за расстояний.

                        Вся эта мышиная возня в рамках сотен и тысяч лет это быстро проходящее. Лучше подождать и посмотреть. Тем более что теория что будет дальше должна быть ими давно написана и проверена на похожих примерах.


              1. Sunrise33g
                28.11.2023 17:50
                +2

                Если никакой информации не давать, а только объявить о существовании развитой цивилизации - и выставить требования, каким нужно удовлетворять для будущего сотрудничества - это с одной стороны, унижение, с другой - вмешательство в чужую цивилизацию с труднопредсказуемыми последствиями.

                Это всё тезисы земных пропагандистов. А точнее пропагандистов тоталитарных режимов, силовики которых могут поддерживать режим только в условиях изоляции населения от других цивилизационных концепций. Изоляционизм - определённо зло.

                Они могут ждать, пока поумнеем.

                Большинство людей никогда не "поумнеют", пока не появятся внешние факторы, значительно снижающие конкурентоспособность "не поумневших" относительно "поумневших".


            1. sim2q
              28.11.2023 17:50

              > Кому представить ценную информацию

              Всем.

              Arrival (2016) - один из любимых фильмов :)
              А по теме - вся информация есть в открытом виде уже давно, но люди в основной массе смотрят не туда.


              1. codecity
                28.11.2023 17:50
                +1

                вся информация есть в открытом виде

                Вы имеете в виду в виде различных теорий заговора типа от Левашова?


                1. sim2q
                  28.11.2023 17:50

                  да ну нет же...
                  надо разобраться для начала кто мы такие, пока копают широким фронтом, кто-то в процессе понимает, причём на любом направлении, но есть прямые объяснения, но там трудно с доказательствами по понятным причинам. Здесь тоже есть статьи - копают в сторону сознания, это уже ближе, но тоже сильно окольный путь.


                  1. p07a1330
                    28.11.2023 17:50
                    +1

                    С таким же успехом можно сказать, что оно уже записано в Вавилонской библиотеке - и это безусловно будет истиной. Но есть нюанс...


                    1. sim2q
                      28.11.2023 17:50

                      Но есть нюанс...

                      заключается в том, что живой процесс идёт и всё происходит:)


            1. anatolykern
              28.11.2023 17:50
              -1

              Всем. 

              С особенностями: каждой из развитых цивилизаций - дать свою и разную информацию (из разных областей).

              И судить о разумности вида на уровне планеты по способности договориться о сотрудничестве и обмене информацией.

              Если смогут договориться, смогут избавиться от паразитирующих элементов (нет смысла договариваться с раковыми клетками), смогут довести баланс технологического и морального развития во временных пределах барьера Ферми без разрушения собственной экосистемы, влияющей и на других разумных на нашей планете - будет иметь смысл и заявляться. Или делать откат.

              Серия статей на https://thegalacticage.substack.com/ достаточно подробно рассматривает разные риски и сценарии.


              1. anatolykern
                28.11.2023 17:50
                +2

                Проявление системы кармы хабра во всей красе. И если я могу понять минус комментарию, как проявление несогласия (хотя стоит его хотя бы обосновывать), то минус профилю с такой аргументацией относится к категории затыкания рта несогласным, на уровне психолого-педагогического развития не "твое действие плохое", а "ты плохой". Спасибо за напоминание о том, почему не стоит тратить время на комментирование на этом ресурсе.
                Проявление системы кармы хабра во всей красе. И если я могу понять минус комментарию, как проявление несогласия (хотя стоит его хотя бы обосновывать), то минус профилю с такой аргументацией относится к категории затыкания рта несогласным, на уровне психолого-педагогического развития не "твое действие плохое", а "ты плохой". Спасибо за напоминание о том, почему не стоит тратить время на комментирование на этом ресурсе.


                1. ShaltaiBoltai
                  28.11.2023 17:50

                  Добро пожаловать на Дыбр!

                  Hidden text

                  Да, рунет напоминает подворотню. Есть, конечно, шанс встретить на его просторах неагрессивного человека, но с 99% вы встретите гопника, который пришел сюда только за тем, чтобы дать кому-то в морду. И даже на общем гопофоне Дыбр выделяется, как колония для малолеток на фоне спального района.

                  Можно, конечно, плюнуть на всю эту гопоту и лишить ее присутствия себя. Но я, например, выбрал другой путь. Пусть они беснуются, минусуют и вообще всячески испражняются. Я пишу не только для них, но и для тех, кто читает и не минусует. От меня не убудет.


            1. ShaltaiBoltai
              28.11.2023 17:50
              -5

              Почему человечество так и не обнаружило другие разумные цивилизации во Вселенной? Наверное потому, что они от нас прячутся?

              Если вы стоите в пустой комнате и кричите "Ау!", а вам никто не отвечает, то, конечно, есть какая-то вероятность, что от вас прячутся. Но, скорее всего, в этой комнате, кроме вас, больше никого нет.


              1. rombell
                28.11.2023 17:50
                +5

                Если вы стоите посреди пустыни и кричите АУ, но никто не отзывается, скорее всего, Вы один на планете.
                Или всё-таки нет?


                1. anatolykern
                  28.11.2023 17:50

                  Узость мышления и ошибочность выводов в обоих комментариях.

                  Вот несколько сценариев сходу:
                  Не слышите ответы или не слышат крики (разные спектры сигналов отправки и восприятия).
                  Заключение о наличии/отсутствии можно делать только в пределах казуальности сигналов и обратной связи.
                  Сигналы просто не интересны, в той же степени как Вам не интересны сигналы ползающих жуков по пустые.
                  Скорость восприятия сигналов может быть совершенно различна (вам дают ответ сигналом длиной в несколько жизней)
                  Забиваете своим криком ответный сигнал.
                  Не прислушиваетесь в достаточной мере к ответному сигналу, который может приходить по другим каналам.


                  1. BugM
                    28.11.2023 17:50

                    Вы не думали что есть некие базовые принципы? С учетом что мы знаем что вселенная одинаковая везде эти принципы тоже неизменны везде.

                    Электромагнитные волны и электромагнитная связь как следствие.

                    Это базовая штука для вселенной. С ее помощью можно наблюдать за удаленными объектами, вселенная довольно прозрачная для них. Связь в пределах планеты или солнечной системы с их помощью тоже очень удобная. Ничего удобнее просто нет. И вероятно не будет. Базовые физические принципы запрещают появление чего-то удобнее. Вероятность что мы эти принципы неверно поняли минимальна.

                    Для любой вообразимой в нормальной научной фантастике цивилизации овладение ими обязательно. Использование их тоже обязательно.

                    Значит логично предположить что будет что-то их слушающее и что-то их посылающее. Как минимум для всех кто не замкнулся у себя и хочет знать что там дальше. Те кто замкнулся нам недоступны и сейчас не интересны.

                    Понятие несущей частоты и модуляции тоже базовое. Как дальше модулировать сигнал могут быть разные варианты. Но у вас будет несущая частота, если вы хотите чтобы ваш сигнал нашли.

                    Слушаем мы все разумные частоты. Есть какие-то краевые случаи, но те же базовые физические принципы говорят что они неудобны. Значит они для всех неудобны.

                    Ну и как итог ловим несущие частоты. Они должны быть. Их нет. Значит или слушаем не то или слушаем плохо или никто не передает ничего достаточно близко. Второе плюс третье на самом деле вероятнее всего. Слушать стоит лучше. Еще более большие телескопы и особенно антенны нужны. Более большие это желательно космических размеров в сотни-тысячи километров.


                    1. rombell
                      28.11.2023 17:50

                      Вот Вы сейчас рассуждаете об обязательности использования лошадей в эпоху, когда уже появились первые автомобили. Очевидно же, что использование гравиволн горазно удобнее для связи на большие расстояния. А ещё удобнее, возможно, использование тёмного вещества или волн тёмной энергии. И то, и другое далеко за пределами наших нынешних возможностей, но мы уже про них знаем.
                      А радиоэфир сильно замусорен


                      1. BugM
                        28.11.2023 17:50

                        Простите, а чем они лучше? Как я и говорил базовые понятия физики мы знаем довольно неплохо. Шанс что мы их неверно понимаем минимален. Что-то вроде гравитационных волн создавать точно сложнее. Условные волны темной энергии будут еще более сложно создаваемы.

                        Никакое развитие не уменьшит расход энергии на их генерацию. Взрыв батарейки это тоже базовый физический процесс. Чем она мощнее, тем сильнее взорвется в случае чего. Использовать на порядки более мощные батарейки там где можно обойтись более слабой странно. Просто так повышается вероятность очень сильных взрывов.

                        Тервер тоже у всех одинаковый. Вероятности могут называться как угодно, но считать их любая цивилизация должна уметь.


                      1. rombell
                        28.11.2023 17:50

                        Радиоволну сложнее создать, чем звуковую, а уж лазерную связь - просто неизмеримо сложнее, чем радио. И тем не менее Человечество перешли сначала на радио, а когда смогли - на оптоволокна. Хотя энергии на все эти рептрансляторы уходит на порядки больше, чем просто крикнуть.

                        Ваши аргументы свидетельствуют только о неразвитости вашей цивилизации, испытывающей проблемы с самыми элементарными потребностями, например, с энергией. Вы даже энергию своей звезды практически не используете!

                        Говорить с вами не о чём. Так, пару зондов закинули в ближайшую звёздную систему и оттуда мониторим.


                1. ShaltaiBoltai
                  28.11.2023 17:50

                  У каждого своя вера. Кто-то верит, что всем управляет ретроградный Меркурий, а кто-то верит, что за ним наблюдают зеленые человечки. Но из-за того, что они с синими человечками в межгалактической ООН подписали хартию о невмешательстве в жизнь примитивных обезьян, мы их не видим и не слышим.


          1. saboteur_kiev
            28.11.2023 17:50
            +4

            Кому представить ценную информацию, ускоряющую поумнение? Китаю, США, России, Европе, Ирану, Израилю?

            А можно нам? =)


          1. AnatolyEmelin
            28.11.2023 17:50

            Ой!


        1. orfelin
          28.11.2023 17:50
          +7

          А чего ждать, если можно помочь поумнеть? 

          может быть по той же самой причине почему на Земле живут изолированные десятки тысяч лет племена, застрявшие в каменном веке и контакт с ними запрещен? Ведь можно было выступить наставниками, дать им железные орудия, антибиотики, нужную информацию, прогрессировать их ? Ан нет - контакты запрещены. И с точки зрения этих племен они единственные человеки на свете.


          1. codecity
            28.11.2023 17:50

            почему на Земле живут изолированные десятки тысяч лет племена, застрявшие в каменном веке и контакт с ними запрещен?

            Понимаю ваше желание как-то сохранить веру в инопланетян и найти логическое обоснование почему они не проявляют себя - но не подходит. Эти племена сами хотят жить в изоляции и ведут себя агрессивно при любой попытке протянуть им руку цивилизации - их выбор. Мы же наоборот изо всех сил ищем контакта и не находим.


            1. kryvichh
              28.11.2023 17:50

              Гипотетическая цивилизация, пережившая цикл сжатия Вселенной, будет настолько отличатся от нашей, что нет смысла даже поднимать подобный вопрос. Практически мы могли бы выйти на контакт с планетарной или звёздной цивилизацией примерно нашего уровня развития, возникшей в соседней звёздной системе после Большого взрыва. Вероятность такого контакта зависит от технического уровня обеих сторон.


              1. codecity
                28.11.2023 17:50

                будет настолько отличатся от нашей, что нет смысла даже поднимать подобный вопрос

                Просто им это может быть не интересно. А так для нас - в любом случае было бы полезно. Даже просто получить ответы на философские вопросы.

                А так - особо нет вопросов, которые мы не способны понять. Что говорить - если даже свое собственное устройство мы хотя и не понимаем до конца, но вопросы ставим и в этом направлении идем.


                1. humminbirdy
                  28.11.2023 17:50

                  С таким же успехом можно предположить, что контакт есть, просто он производится точечно, с организованными системами, вроде лидеров стран или семей, которые обладают большинством ресурсов в мире и фактически курируют развитие экономики и политики. Так что никто не может сказать точно, нет ли уже сейчас контакта, просто нас не ставят в ведение, так как мы фактически не имеем особого влияния на вектор развития. Мы можем вносить свой вклад в науку и культуру обособленно. Фактически сейчас в мире вообще нет причин для вмешательств, если смотреть с точки зрения сохранения существования цивилизации.

                  Так, что, возможно, инопланетяне уже смотрят кино с нашим участием.


                1. rombell
                  28.11.2023 17:50
                  +4

                  >особо нет вопросов, которые мы не способны понять

                  Примерно так же могли бы рассуждать муравьи, если бы могли рассуждать. Самоуверенность, базирующаяся на том, что не приходилось сталкиваться с действительно большими задачами.

                  Например, я точно знаю, что не смогу понять доказательство Великой теоремы Ферма за отпущенное мне время. Просто не успею выучить необходимые области знания (я не математик). Предполагаю, что людей, могущих сходу разобраться в доказательстве, не тратя на это годы, в мире считанные десятки человек.

                  И я легко могу экстраполировать на теорему со сложностью доказательства на порядок выше - и всё, уже не найдётся людей, могущих это доказательство осознать. Например, понадобятся настолько разные области математики и настолько сложные связи, что просто не получится одновременно оперировать всем сразу.


            1. XaBoK
              28.11.2023 17:50
              +2

              Я думаю вы сосредоточились на текущей точке, но с точки зрения древней цивилизации мы, скорее всего, не достигли уровня "разговора". Мысленный эксперимент: с рождения первых разумных людей с шагом в 100 лет, сколько прошло бы итераций "контакта" для принятия концепции инопланетной разумной жизни? Ну или хотя бы сколько итерации приняли бы концепцию без божеств и идолов? Можно посмотреть, сколько понадобилось для шарообразной Земли, вращающейся вокруг Солнца. Хотя, ой ... Теперь добавим к этому выбор тех, с кем из Землян в этих итерациях стоило говорить.

              Ну не муравьи, но и не те, кто могут хотя бы понять в общих чертах. Это как объяснять фондовый рынок 2-х летнему ребенку. Я бы не стал. Подрастёт - поговорим. В идеале - сам спросит.


              1. ShaltaiBoltai
                28.11.2023 17:50
                +1

                Фондовый рынок объяснить не так уж и сложно. Фондовый рынок - это место, где меньшинство обогащается за счет обеднения большинства, добровольно отдающего свои деньги.


          1. saboteur_kiev
            28.11.2023 17:50
            +1

            может быть по той же самой причине почему на Земле живут изолированные десятки тысяч лет племена, застрявшие в каменном веке и контакт с ними запрещен?

            У них просто нефти нет. А так никто не запрещает с ними контактировать, кроме них самих.


            1. MishaRash
              28.11.2023 17:50
              +1

              Вроде бы есть и другая причина: для изолированных популяций крайне опасны инфекции, пусть даже безобидные для активно связанной части человечества.


              1. saboteur_kiev
                28.11.2023 17:50

                Поэтому вирус Эбола в африку едут лечить европейцы и американцы, а не они как-то сами?


                1. MishaRash
                  28.11.2023 17:50

                  А Эболой болеют именно подчёркнуто изолированные племена? Что-то не похоже, хотя я точно не знаю.

                  Я имел в виду видео про катастрофические для американских индейцев болезни, которое смотрел недавно. Там, кстати, отсутствие настолько ужасных последствий в Евразии связывается с передачей болезней от животных и обратно и тем, что в Америках их было одомашнено меньше. Таким образом, наверное, и изолированные популяции могут получать страшные болезни, но далеко не все, и одна из "общих" может оказаться для них неожиданно тяжёлой. Хотя могу ошибаться.


            1. ShaltaiBoltai
              28.11.2023 17:50

              :-)

              Ну, отсутствие или наличие нефти не очень надежный показатель. В мире как раз больше народов, у которых нефть есть, но они, тем не менее, как и северные сентинельцы, тоже всеми конечностями отбрыкиваются от внешнего мира.


          1. ShaltaiBoltai
            28.11.2023 17:50

            Насколько мне известно, контакты запрещены только в отношении одного единственного племени - знаменитых северных сентинельцев. О других подобных запретах я не слышал. И даже запрет посещать сентинельцев не останавливает индивидуумов, желающих стать обедом. Так что, запрет, не сказать, что уж очень и строгий.

            И вы ошибаетесь насчет того, что "эти племена" считают, что "они единственные человеки на свете". 200 лет назад вполне могли существовать племена, которые были не в курсе, что где-то еще живут такие же обезьяны, как и они. Но сейчас таких племен точно нет. Даже самые изолированные из них - те самые северные сентинельцы прекрасно в курсе, что они не единственные жители в этом мире. Скорее всего, они даже в курсе, что весь остальной мир живет куда более развито т.е., комфортно, чем они. Но они вот не хотят. Прям, как мы.


        1. RMV1983
          28.11.2023 17:50

          А чего ждать, если можно помочь поумнеть?

          Ну, так может они так и делают. Но с учётом размера Вселенной и кол-ва цивилизаций, наша очередь наступит скажем лет эдак примерно через 10^10.


        1. selivanov_pavel
          28.11.2023 17:50

          А чего ждать, если можно помочь поумнеть?

          Зачем им это? Вы же не бросаетесь на помощь каждому пробегающему мимо таракану. Какие могут быть причины для контакта, что он им даст?


        1. ginya
          28.11.2023 17:50
          +1

          А может быть они уже, только вот "Трудно быть богом".


        1. Tulazerg
          28.11.2023 17:50

          Простите. Но это и так происходит. То,что вам об этом не известно, не значит, что этого нет.

          В пример, могу привести Николаса Теслу, который фактически изобрёл современное электричество и способы его добычи и передачи.

          Почитайте о том, как он получал информацию, он сам об этом не стеснялся говорить. Про его сны и тд.

          Технология современного компьютера, основанная на двоичном коде - тоже дана "извне".

          Как и многое другое.

          Эти технологии не Путину, Обаме или Си даются. А, конкретным людям, обычно учёным, не в интересах которых говорить, что им просто инопланетяне дали инфу, ходя даже из этого правила есть исключения.


          1. kryvichh
            28.11.2023 17:50

            Надо понимать как работает мозг. Днём мы накапливаем информацию, а ночью она "раскладывается по полочкам". Сон лишь побочный эффект этого процесса.


      1. Spaceoddity
        28.11.2023 17:50

        Гравитация, гравитация... Два момента интересуют:

        1. Тезисно - какое положение в вашей концепции занимает понятие "времени" (временной оси)? Эта сущность оторвана от сущности вселенной? Или является её частью? Грубо говоря, вот эта пульсация Вселенной происходит на одной временной оси, или каждый Большой взрыв формируется новая временная ось?

        2. Что насчёт квантовой запутанности? Могут ли цивилизации "бэкапить" себя таким образом (реликтовое холодное пятно)?


        1. Gorkavyi Автор
          28.11.2023 17:50

          Время всегда одно для всех циклов, ничего нового я в понятие времени не вношу - все как в ОТО. Ничего не могу сказать про космологическую роль квантовой запутнности - не специалист.


      1. gans_2
        28.11.2023 17:50

        А какой смысл вообще вступать с такими ленивыми и не любопытными существами?

        Мы даже не смотрим никуда ни через гравитационную линзу Солнца ни через рефракционные линзы планетных атмосфер.

        Доклад на семинаре НКЦ SETI им. Л. М. Гиндилиса совместно с секцией "Жизнь и разум во Вселенной" Научного совета по астрономии РАН. 24 февраля 2023г с уточнениями Панова Александра Дмитриевича(НИИЯФ МГУ, доктор физ.-мат. наук, ведущий научный сотрудник)


    1. commanderkid
      28.11.2023 17:50

      Ну, с учётом того, что жизнь возникла СРАЗУ после образования Земли.. . Есть теория, что жизнь возникла около 8-10 миллиардов лет назад, по анализу усложнения генетической кода.... Тогда температура вселенной была 30-40 градусов цельсия, что позволяло существовать жидкой воде. Подробней: https://youtu.be/-B_0Z8Ueo6g?si=miUIcN_FqXCR9JA0


      1. edo1h
        28.11.2023 17:50

        температура вселенной? жидкая вода в космосе?


        1. ksbes
          28.11.2023 17:50

          Ну сейчас температура вселенной 3 Кельвина, а тогда была 300 (всем молчать про тракториста!)
          Другое дело что тогда воды быть не могло, т.к. кислорода ещё не было ...


          1. johnfound
            28.11.2023 17:50

            Чтобы вода была жидкой, нужна не только температура но и концентрация. У меня в комнате вода в воздухе точно есть, но концентрация недостаточна чтобы была жидкой.


            1. ksbes
              28.11.2023 17:50

              Не столько концентрация, сколько давление. На Луне скафандр в тазике не простираешь!
              Но 8-10 млрд лет назад существовали всякие скопления материи, где давления было достаточно (ну вроде как сейчас в определённых слоях атмосферы газовых гигантов). Но там было явно недостаточно металличности для возникновения жизни.


    1. DaneSoul
      28.11.2023 17:50

      Цивилизации пережившие цикл, очевидно должны были успеть заселить всю вселенную. И где они? (С)

      Может наша Земля для них что-то типа аквариума/террариума в котором прикольно для развлечения наблюдать как мелочь всякая копошится?


      1. anatolykern
        28.11.2023 17:50

        Скорее заповедная зона с запретом прямых контактов и уникальными условиями для возникновения биологической жизни.


        1. ksbes
          28.11.2023 17:50
          +2

          Скорее Землю ещё найти надо среди миллиардов триллионов звёзд.


    1. PrinceKorwin
      28.11.2023 17:50

      Цивилизации пережившие цикл, очевидно должны были успеть заселить всю вселенную. И где они? (С)

      А может быть человечество и есть такая цивилизация, только при прохождении через цикл что-то пошло не так и мы обнулились. Как это доказать или опровергнуть? :)


      1. MishaRash
        28.11.2023 17:50

        Ну если постулировать, что очень уж качественно обнулились, то опровергнуть никак, и это уже не научный тезис.

        Нравится ещё "очевидно" про цивилизации на совершенно другом уровне развития, как будто может быть понятна их мотивация и всё такое.


    1. korva
      28.11.2023 17:50

      Условно, как мне, необразованному тупому быдлу, кажется - ничто не мешает нашу 3d-вселенную свернуть в кольцо в более высоком измерении по одному направлению, или нескольким, и получить бесконечную вселенную-кольцо. Как двумерную линию на трубке из трёхмерного пространства, которая начинается и кончается в одной точке, обогнув свёрнутый лист и встретившись с исходной точкой на стыке. И бегай по кругу до потери пульса.


  1. wmlab
    28.11.2023 17:50
    +1

    А хотя бы приблизительное расстояние от нас до центра Вселенной эта теория может оценить? Он ближе или дальше горизонта событий?


  1. Gorkavyi Автор
    28.11.2023 17:50
    +1

    Между краем расширяющейся Вселенной и ее центром, видимо, 46 миллиардов световых лет. Но где мы в этом пространстве - сказать трудно, не исключено, что примерно на краю, в главной волне расширения.


    1. ginya
      28.11.2023 17:50

      Я понимаю, как при возрасте 13,8 млрд лет получается расстояние в 27,6 млрд св лет. А откуда 46 миллиардов лет, тем более радиус, а не диаметр?


      1. bromzh
        28.11.2023 17:50
        +1

        То есть возраст вселенной не 13,8, не 26,7, а целых 46 миллиардов лет?

        Нет. Из-за того, что вселенная расширяется, её размер в световых годах может быть больше её возраста

        Но про возраст я бы тоже хотел спросить. @Gorkavyi, ваша теория что-то говорит о возрасте текущей вселенной? Просто в различных новостях проскакивают иногда новости, что данные от джеймса вебба иногда приходят странные, видны галактики, которые не могли бы так рано образоваться. И как будто возраст вселенной может быть больше. Это как-то влияет на теорию, может ли её как-то опровергнуть или подкорректировать?

        PS не успел полностью прочесть вашу книгу, мб там есть ответ?


        1. Gorkavyi Автор
          28.11.2023 17:50
          +1

          Текущая Вселенная прекрасно вписывается в циклическую космологию, как часть цикла, начавшегося от максимального сжатия. Галактики, "которые не могли бы так рано образоваться" в инфляционной теории, прекрасно образуются в циклической космологии благодаря сверхмассивным черным дырам, которые начинают формировать вокруг себя аккреционные диски - протогалактики еще на первых миллионах лет, когда среда вокруг очень плотная. Это рассмотрено и в книге и в статье [7] из списка выше.


  1. adeshere
    28.11.2023 17:50

    Подскажите, пожалуйста:

    1. В момент максимального сжатия плотность энергии излучения будет огромной. Какая часть прироста общей массы ЧД обеспечивается поглощением излучения (а не слиянием ЧД), и как она меняется от цикла к циклу?

    2. Могут ли возмущения гравитационного поля в тот же момент заметно влиять на эффекты вблизи горизонта событий ЧД (с учетом их вращения)? Например, приводить к столь сильному росту излучения Хокинга, что общая масса ЧД на каком-то цикле перестанет заметно расти? Или там градиенты слишком малы для этого?

    3. Что происходит с горизонтом событий СМЧД в столь неоднородном внешнем поле (в момент максимального сжатия)? Как он меняется, и влияет ли это на эффективное сечение рассеяния?

    P.S. Прошу прощения, если в книжке есть ответы на эти вопросы. Я пока только рекомендованный научпоп прочитал. Боюсь, что пока я осилю книжку, тут разговор заглохнет..


    1. adeshere
      28.11.2023 17:50

      Вдогонку - все вопросы про сверхмассивные ЧД


    1. Gorkavyi Автор
      28.11.2023 17:50
      +1

      1. "В момент максимального сжатия плотность энергии излучения будет огромной. Какая часть прироста общей массы ЧД обеспечивается поглощением излучения (а не слиянием ЧД), и как она меняется от цикла к циклу?"

      100%, ведь слияние дыр не увеличивает общую массу популяции ЧД, а даже уменьшает. Если говорить об росте индивидуальной массы ЧД, то она зависит от размера ЧД и там есть прирост и от излучения и и от поглощения более мелких дыр. Эти процессы повторяются от цикла к циклу без существенных изменений.

      1. "Могут ли возмущения гравитационного поля в тот же момент заметно влиять на эффекты вблизи горизонта событий ЧД (с учетом их вращения)? Например, приводить к столь сильному росту излучения Хокинга, что общая масса ЧД на каком-то цикле перестанет заметно расти? Или там градиенты слишком малы для этого?"

      Излучение Хокинга для черных дыр звездных масс пренебрежимо мало,

      1. "Что происходит с горизонтом событий СМЧД в столь неоднородном внешнем поле (в момент максимального сжатия)? Как он меняется, и влияет ли это на эффективное сечение рассеяния?"

      Это интересный и сложный вопрос. Плотность СМЧД приближается к плотности среды в момент максимального сжатия, и вопрос в том, как определить в этом случае понятие дыры - в условиях, когда все наблюдатели падают и свободно проникают в любые дыры, а сами дыры стремительно растут и из вещество становятся сердцевинами более крупных дыр.


      1. adeshere
        28.11.2023 17:50

        100%, ведь слияние дыр не увеличивает общую массу популяции ЧД, а даже уменьшает.

        Виноват, криво спросил...

        Я имел в виду рост массы СМЧД. Считая излучение и обычные ЧД ЗМ за "корм".

        Ведь на начальных циклах СМЧД еще не было? Были только ЧД звездной массы? Но наиболее крупные из них росли быстрее остальных, причем чем больше масса - тем быстрее, в результате чего через много-много циклов появились СМЧД? Я правильно понял идею?

        Вопрос был в том, притормаживается ли рост СМЧД каком-то этапе (цикле), или процесс только ускоряется? Ведь если рост СМЧД идет с ускорением по мере набора массы, то тогда получается, что начальная стадия роста должна включать очень много циклов, а вот на современном этапе, когда массы СМЧД на порядки отличаются от обычных ЧД - все уже должно пойти в разнос, и "наша" стадия завершится за гораздо меньшее число циклов. В силу чего вероятность ее застать очень маленькая. Снова упираемся в антропный принцип...

        Из вот этой статьи я понял, что сперва (на первых циклах) у нас есть только ЧД звездной массы, причем в небольшом количестве. Соответственно, для возникновения эффекта "отскока" (механизм которого связан со взаимодействием ЧД) Вселенная должна сжаться до очень маленького радиуса, т.к. ЧД еще мало, и они маленькие, а нам нужно, чтобы они бурно взаимодействовали. В следующем цикле количество ЧД увеличивается, так как к "реликтовым" ЧД добавляются вновь образовавшиеся, и так далее. По мере роста количества и массы ЧД радиус минимального сжатия будет увеличиваться (или нет?!), а плотность энергии излучения в этот момент - уменьшаться (так как суммарная масса излучения уменьшается за счет роста ЧД, а объем растет). Что вроде бы должно привести к изменению соотношения типов "корма" у СМЧД.

        Если бы вдруг оказалось, что крупнейшие ЧД растут преимущественно за счет излучения (а не других ЧД), то на более поздних циклах их скорость роста СМЧД может стабилизироваться, несмотря на рост массы. Благодаря этому эволюция могла бы пойти не по экспоненте, а более спокойно, и эта стадия заняла бы больше циклов. И, соответственно, шансы застать "наш" мир повышаются.

        Или я все совсем неправильно понимаю?

        Я ведь совершенно не физик (учился на физфаке, но очень давно и с тех пор с ЧД ни разу не сталкивался). Поэтому для меня это сугубо философский вопрос... но к которому все-таки хочется иметь какое-то простое (иллюстративное) объяснение...

        Если говорить об росте индивидуальной массы ЧД, то она зависит от размера ЧД и там есть прирост и от излучения и и от поглощения более мелких дыр. Эти процессы повторяются от цикла к циклу без существенных изменений.

        Наверно, мне надо сперва дочитать книжку до конца, так как я явно где-то туплю. Если все повторяется без существенных изменений, значит количество и спектр масс ЧД на начальных циклах были такие же, как сейчас?

        Получается, я ничего не понял в модели?

        Излучение Хокинга для черных дыр звездных масс пренебрежимо мало

        В однородной среде - разумеется. Я хотел уточнить, не может ли хаос ГВ от расположенных в паре шагов источников что-нибудь изменить. Рассуждая логически - вроде не должен (поля экстремальные, но градиенты не те). Но бытовая логика в таких вопросах часто подводит, поэтому были сомнения. Извините, если вопрос совсем глупый...

        Плотность СМЧД приближается к плотности среды в момент максимального сжатия

        Меня всю жизнь пугал этот кошмар: когда плотность материи в какой-то области сначала чуть меньше критической, а потом прилетает малое возмущение, и для одного наблюдателя она стала больше критической, а для другого, расположенного рядом, - не стала. Вот и Вы тоже простыми словами не можете объяснить... Получается, тут как в квантмехе, интуитивно понятные аналогии типа горизонта событий полностью бесполезны, а единственный выход - это "заткнись и считай"?


        1. Gorkavyi Автор
          28.11.2023 17:50
          +1

          Всю эволюцию Вселенной можно разделить на две части - начальную эволюцию, при которой разные компоненты Вселенной меняли свои массы и взаимоотношения, и стационарное состояние - когда Вселенная пришла к настоящему виду. В стационарном состоянии циклы повторяются практически идентичными. Именно к этой стадии относится мое утверждение: Если говорить об росте индивидуальной массы ЧД, то она зависит от размера ЧД и там есть прирост и от излучения и и от поглощения более мелких дыр. Эти процессы повторяются от цикла к циклу без существенных изменений.

          В начальной стадии, когда Вселенная эволюционировала, предположим, из начального состояния в виде разреженного газа, ей понадобилось много времени, чтобы накопить темну материю и прийти, например к СМЧД, Эта начальная эволюция практически не исследована - и там конечно может быть изменение радиуса максимально сжатой Вселенной. Всё, что мы могли показать с Тюльбашевым: если взять начальный набор чд звездной массы, которые образовались за один цикл, и посмотреть их рост при постоянной среде питания (что, как вы правильно заметили, упрощение, потому что при переходе части вещества и излучения в дыры, плотность питающей среды должна как-то меняться), то получится, что такие дыры растут очень медленно и им надо сотни циклов, чтобы добраться по средней массе до 100 масс солнца. Это и позволяет накопить темную материю из таких дыр (ведь на каждом цикле их число прибавляется за счет новорожденных). После 100 масс солнца эволюция ускоряется и СМЧД стремительно формируются в ходе последнего из рассмотренного цикла. Итак, мы оценили, что приход к нынешему состоянию Вселенной должен занять около 1000 циклов (это не считая количества циклов, которые привели к созданию ЧД звездных масс). Притормозить рост СМЧД вряд ли можно - их судьба слиться в Большую Черную Дыру при максимальном сжатии Вселенной, а те СМЧД, которые остались, разбросать - именно вокруг них возникнут галактики.


        1. Gorkavyi Автор
          28.11.2023 17:50

          Вопрос неглупый, но на него не ответишь без расчетов.

          Я не думаю, что всплеск плотности может по разному восприниматься разными наблюдателями. Если есть всплеск выше критической плотности, то будет ЧД без вариантов. Так как обычные ЧД очень массивны, то квантовыми парадоксами можно пренебречь.


        1. korva
          28.11.2023 17:50

          Я вот имел дело со взрывчаткой в молодости и читал учебники по криминалистике и исследованию продуктов взрыва. Кратко - даже такие чувствительные вв как тэн не полностью реагируют. Остаются ничтожные остатки. Как знать - быть может смчд это ошметки исходной сингулярности?


          1. Gorkavyi Автор
            28.11.2023 17:50

            Я согласен, что ошметки в виде СМЧД будут, только не от сингулярности, от которой мы избавились, а от первоначального вселенского шара в 10 светолет.


  1. inakrin
    28.11.2023 17:50
    +1

    Сравнивали ли вы качество перевода от Deepl и Chatgpt? По моему опыту deepl выдаёт значительно качественней перевод, но могло измениться всё уже с тех пор, как я его использовал последний раз.


    1. janvarev
      28.11.2023 17:50
      +1

      У меня есть большой бенчмарк по сравнению переводчиков: https://github.com/janvarev/OneRingTranslator/blob/main/docs_md/ESTIMATIONS.md - и там же опенсорсная прога с помощью которой можно переводить и делать сравнения.

      Так да, Deepl лучше почти всех (но кстати Яндекс еще лучше по метрикам)


    1. Gorkavyi Автор
      28.11.2023 17:50

      Нет, не сравнивал.


    1. Gorkavyi Автор
      28.11.2023 17:50

      Ну вот отзывы 4 рецензентов на статью по энтропии (см. добавленный P.S. в статье) - про физику спорят, а английский текст, сгенерированный ChatGPT (c моей редакцией) - полностью одобряют.


  1. sim2q
    28.11.2023 17:50
    +1

    Не люблю замкнутых пространств, поэтому бесконечная инфляция ("кипящая" вселенная) по А.Линде мне ближе :)

    потому что на одного умного собеседника, которые задает хорошие вопросы, находится с десяток буквальных гениев

    Если что я из "этих", так что просто поделился переживанием:)


  1. smrl
    28.11.2023 17:50
    +12

    Предсказание циклической космологии о реликтовом фоне низкочастотных гравитационных волн, вызванных массовым слиянием черных дыр при сжатии Вселенной до минимального размера в десять световых лет, уже подтверждено: консорциум NANOGrav недавно обнаружил этот фон наногерцовых волн.

    Это, конечно, замечательно (особенно если все члены консорциума согласятся с тем, что эксперимент делался дла проверки уникальных предсказаний вашей теории, несовместимых ни с одной другой). Но все-таки: а какие наблюдения (при совеременном развитии астрономии) могли бы однозначно опровергнуть вашу теорию?


    1. Gorkavyi Автор
      28.11.2023 17:50
      +5

      Эксперимент по поиску наногерцовых волн шел сам по себе долгие годы, накапливая данные. Мы с Тюльбашевым, рассматривая популяцию черных дыр во Вселенной, предсказали, что должен существовать сильный фон гравитационных волн, вызванных слияниями черных дыр звездных масс. Это предсказание сразу задает типичный диапазон частот реликтовых гравитационных волн. Именно в этом диапазоне и были найдены наногерцовые волны наблюдателями, что мы резонно расцениваем, как подтверждение нашей теории. Какие наблюдения могут опровергнуть нашу теорию? Ну, например, если будут найдены элементарные частицы, которые обеспечивают 100% темной материи. Или будет найдена кватовая сила, которая вызывает Большой Взрыв и существующее ускорение Вселенной.


    1. MishaRash
      28.11.2023 17:50
      +1

      особенно если все члены консорциума согласятся с тем, что эксперимент делался дла проверки уникальных предсказаний вашей теории, несовместимых ни с одной другой

      Насчёт цели эксперимента не совсем релевантное замечание. Наоборот, намного интереснее, когда получаются незапланированные открытия.

      При этом не возражаю насчёт вашего общего скепсиса. Для наногерцовых волн есть множество других пояснений, насколько я знаю. Так что вопрос о наличии более уникальных подтверждённых предсказаний очень уместен.


      1. Gorkavyi Автор
        28.11.2023 17:50
        +1

        В статье [6] из вышеприведенного списка делается дополнительное предсказание о наногерцовых волнах - о резком падении мощности гравволн для периодов меньше года. Все другие модели не видят в этом интервале ничего особенного. В книге - в параграфе 20.3 з- делается предсказание, что самой большой энергией будут обладать килогерцовые гравволны. Тоже от слияния черных дыр звездных масс, но только генерируемых на стадии не минимального, а максимального размера Вселенной. Хотя таких слияний немного, но у них самое бонусное красное смещение, даже синее.


  1. delimer
    28.11.2023 17:50
    +1

    А как при этом ведёт себя энтропия? Как она меняется при осциляциях?


    1. adeshere
      28.11.2023 17:50
      +1

      Энтропия была первоначально введена, как макропараметр для идеального газа - системы с упругими соударениями, без диссипации и т.д. Там все прекрасно обосновано эмпирикой. Но потом ее стали обобщать на такие системы, где исходное определение непригодно. Я еще понимаю подход на основе числа возможных состояний, который логически согласуется с термодинамикой (стрела времени и т.п.). Даже готов принять, что такой подход работает в системе (газе) с химическими реакциями. Ну а как насчет ядерных? С улетающими (прилетающими) нейтрино? Где тут граница замкнутой системы? Там, где мы ее мысленно проведем? Вот есть сверхновая в сферическом вакууме; объясните мне, как меняется ее энтропия в процессе? Только без непроверяемых допущений, а из первых (экспериментально доказанных) принципов? Как вообще правильно посчитать энтропию системы с гравитацией (которая пока не очень квантуется)? Сколько всего состояний у ЧД, и как меняется энтропия в процессе коллапса?

      В общем, я не понимаю, почему все уверены, что разные способы подсчета энтропии тождественны в общем случае, и что закономерности, доказанные в рамках одного из определений, будут верны для других (особенно для нетривиальных систем с сильным взаимодействием между элементами, включая их слияние/разделение). Всегда ли система с меняющимся числом элементов обладает всеми термодинамическими свойствами замкнутой? А если это космологическая система в бесконечном пустом пространстве? Например, у нас был "идеальный газ" из ЧД (неплотный, ньютоновский, т.е. соударения упругие), а потом он сжимается и начались столкновения со слиянием, излучением ГВ и т.д.?

      Никаких экспериментальных подтверждений, что мы вправе экстраполировать классическую энтропию на такие системы, как осциллирующая Вселенная, имхо не видно. Есть только попытки обобщить ее так, чтобы расширенное толкование более-менее согласовывалось с экспериментом. Но с чего мы решили, что это натянутое на глобус обобщение и правда экстраполабельно?

      Да, общепринятый научный метод основан на экстраполяции наблюденных закономерностей в более широкие области. Но именно в случае с энтропией это уж очень сильно смахивает на подгонку. Лично у меня очень большие сомнения в предсказательной силе таких обобщений.


      1. ksbes
        28.11.2023 17:50
        +2

        Ну вообще-то работать с термодинамическими потенциалами для почти замкнутых систем умеют довольно давно. Иначе даже ДВС не смогли бы рассчитать. А это довольно не замкнутая система в целом.
        С теми же нейтрино: рассматриваем окружение как абсолютно чёрное тело температуры ноль. Как говориться - улетело, так улетело. И это не единственный способ.

        Но когда говорят об энтропии квантовой и всяких чёрных дыр - то там речь идёт по сути об энтропии информационной и с термодинамической она связанна косвенно. И так как у нас нет понятия информационной(квантовой) температуры и информационной(квантовой) внутренней энергии и т.п. то речь о термодинамики собственно не идёт.
        И соответственно ограничения области применения термодинамики на такое понятие энтропии уже не распространяются. И даже закон неубывания энтропии получается шире чем второй закон термодинамики (этот вообще про поток энергии, а не про энтропию так-то). И, так же как и закон сохранения энергии - в рамках ОТО не строгий.

        Так что никто не экстраполирует классическую энтропию, а пользуются новым определением, которое даже размерность другую имеет (биты против Джоуль на Кельвин). Всё в порядке с этим!


        1. Gorkavyi Автор
          28.11.2023 17:50

          "Но когда говорят об энтропии квантовой и всяких чёрных дыр - то там речь идёт по сути об энтропии информационной и с термодинамической она связанна косвенно. И так как у нас нет понятия информационной(квантовой) температуры и информационной(квантовой) внутренней энергии и т.п. то речь о термодинамики собственно не идёт".

          Это не так. Для черных дыр определена температура (связанная с излучением Хокинга), поэтому там энтропия вполне имеет термодинамическую трактовку, в которой черные дыры рассматриваются как экстремально холодные тела с огромой энтропией.


          1. ksbes
            28.11.2023 17:50

            Для термодинамического определения энтропии одной температуры мало. Нужна ещё и внутренняя тепловая энергия (не путать с просто энергией и свободной энергией). Причём не просто внутренняя энергия, а её зависимость от температуры вплоть до абсолютного нуля (иначе вы абсолютную энтропию не подсчитаете).
            Соответственно ничего этого для чёрной дыры не определено. А если попытаться определить хоть как-то то получим чушь: из-за особенности излучение Хокинга температура чёрной дыры падает при увеличении "энергии" - т.е. тепловая энтропия вообще отрицательной получается.
            Так что когда говорят об энтропии черной дыры имеют ввиду либо информационное, либо очень на него похожее квантовое определение.


            1. Gorkavyi Автор
              28.11.2023 17:50

              Я не переворачиваю существующие и достаточно общепринятые представления Хокинга-Бекенстейна о термодинамике черных дыр, а показываю, как можно избавиться от термодинамического тезиса Толмена против модели циклической Вселенной. И интерпретация энтропии ЧД не очень существенна.


    1. Gorkavyi Автор
      28.11.2023 17:50
      +1

      Рис. 8 Циклическая эволюция: радиуса пульсирующей Вселенной (a), галактик и размера Большой Черной Дыры (b) и энтропии наблюдаемой части Вселенной (c). В качестве начала цикла берется не Большой взрыв, а поглощение поля галактик растущей Большой Черной Дырой, из-за чего энтропия наблюдаемой Вселенной падает на много порядков. Черные дыры, растущие в сжимающейся Вселенной, увеличивают энтропию мира. При максимальном сжатии (перед моментом Большого взрыва) энтропия растет скачком из-за образования начальной большой дыры. Рост черных дыр, включая большую дыру, в расширяющейся Вселенной снова увеличивает глобальную энтропию.

      Вот так меняется энтропия во Вселенной (раздел 13.1 в книге). Падение энтропии - это только для наблюдаемой части Вселенной. С точник зрения глобального наблюдателя, второй закон термодинамики не нарушается и поверхность Большой Черной Дыры уносит основную энтропию с собой, но ее судьба нас не должна волновать.


    1. Gorkavyi Автор
      28.11.2023 17:50

      Сегодня вышла в препринтах моя работа про энтропию Вселенной (см. добавленный P.S. в статье)


  1. AlekseiMorozov19730316Ru
    28.11.2023 17:50
    -8

    Суть космологической революции XXI века - смена модели одноразовой непрочной Вселенной, тлеющие обломки которой разлетаются по бесконечной холодной пустыне, на циклическую космологию, которая описывает вечную Вселенной, пульсирующую внутри огромной черной дыры.

    :) Ну, "хрен редьки не слаще". Попытка смешивания "нелепостей бигбэнгства", "ахинеи чернодырства" и "абракадабры эйнштейнианства" - это не "революция", а просто "обострение хронического псевдонаучного галлюциноза". Всё подобное "творчество" легко опровергается Эфирной Теорией Всего и скоро будет выброшено на помойку науки...


    1. bromzh
      28.11.2023 17:50
      +3

      Да, ведь настоящая наука она не там, в институтах, лабораториях, журналах, а в комментах на хабре с картинками кубов, да

      скоро будет выброшено на помойку науки

      "он наступит скоро, надо только подождать"


  1. dolbi
    28.11.2023 17:50

    Все больше исследований движется в сторону понимания природы цикличной Вселенной. Что не может не радовать. Самому интересна эта тема. Разве что.

    Разве что, наверное, не совсем могу принять идею осциллирующей Вселенной в черной дыре. Или осциллирующей ЧД во Вселенной? Все же ближе мне гипотеза, что Вселенная - мембрана, находящаяся в балке. Где она как раз развернута в виде тех 3+1 измерений, в которых мы живем. А периоды расширения/сжатия есть ничто иное как нарастающая и ниспадающая энтропия.


  1. Firsto
    28.11.2023 17:50

    Вот стало интересно, в каждый следующий цикл вселенная разворачивается одинаково или получающаяся материя будет занимать разное положение или может какие-то константы могут быть другими?


    1. Gorkavyi Автор
      28.11.2023 17:50
      +1

      Примерно одинаково, но константы точно те же.


  1. johnfound
    28.11.2023 17:50
    +1

    У меня есть единственный вопрос к современной физике: «Почему материя движется?». Вопрос на котором обычный ответ: «Движение внутренне-присущее свойство материи». И этот ответ меня не удовлетворяет, так как по сути он означает «Мы не знаем. Все движется просто так само собой.»

    А вопрос настолько фундаментальный, что нет вопросов фундаментальнее. От ответа возможно зависит вся теоретическая физика.

    Смотрите, если у какой-то частицы есть координаты и они меняются, физика прекрасно описывает первую производную этих координат – скорость: (хоть Ньютон, хоть СТО), вторую производную – силу (опять и Ньютон и OTO и всякие теории поля).

    Для изменения скорости нужна сила (ускорение). А если сила отсутствует, то координаты все-равно меняются беспричинно. Почему для изменения первой производной мы видим причину (силу) а для изменения координат нет? А может причина есть но мы ее не видим? А если нет такой причины, но это должно как-то быть обосновано. Объяснение "все на самом деле так" на обоснование не тянет.


    1. codecity
      28.11.2023 17:50

      И этот ответ меня не удовлетворяет, так как по сути он означает «Мы не знаем.

      Т.е. ваш вопрос сводится к следующему: что мы можем принять за аксиому, за явления фундаментальные - а что таковым не является. К примеру, то что материя в принципе существует, а могла бы не существовать - с этим вы смирились и готовы принять без доказательства.


      1. johnfound
        28.11.2023 17:50

        Движение слишком сложный феномен, чтобы принимать его в качестве аксиомы.

        А вот существование материи, да, можно. Заметьте, что такое допущение никак не ограничивает природу материи. Хоть волны неведомого поля, хоть шарики из глины следующего уровня Вселенной, хоть числа в памяти вселенского компьютера.


        1. ksbes
          28.11.2023 17:50

          Для начала определитесь (т.е дайте аксиоматическое определение) что такое "координаты". Т.е. материя существует - это хорошо, но что такое "материя существует в этой точке в это мгновение"?

          И ваши рассуждения вас проведут по проторенной дороге Галлилея, Ньтона и Ко - и соответственно получите принципы относительности Галлелея (про которые так любят забывать сейчас), из которых прямо следуют законы Ньютона (в исходной форме)


          1. johnfound
            28.11.2023 17:50

            Из принципа относительности Галилея (да и Эйнштейна) никак не следует что абсолютные координаты и абсолютное движение не существуют. Из них следует только то, что мы или не видим эту абсолютную систему отсчета или она никак не отличается от остальных, то есть мы ее не можем отличить, хоть и видим.

            Хотя, в статьях о космологии, ученые сплошь и рядом используют именно абсолютные координаты, хоть и явно это не объявляют, потому что заклюют. Например скорости измеренные относительно реликтового излучения вполне тянут на абсолютные скорости.

            Но ладно, примем, что абсолютные координаты вообще не существуют. Тем не менее, то что тела объективно меняют свои координаты в пространстве можно доказать запросто: Берем две тела, которые движутся относительно друг друга – это явление объективно существует и просто наблюдается. Чтобы они двигались, то координаты в пространстве должны меняться хоть у одного из тел. Или у обоих. Ситуация в которой координаты не меняются у обоих тел возможна только если они не движутся относительно друг друга. Что противоречит с условием задачи.


            1. MishaRash
              28.11.2023 17:50

              Из принципа относительности Галилея (да и Эйнштейна) никак не следует что абсолютные координаты и абсолютное движение не существуют. Из них следует только то, что мы или не видим эту абсолютную систему отсчета или она никак не отличается от остальных, то есть мы ее не можем отличить, хоть и видим.

              Чем та система отсчёта тогда такая абсолютная (выделенная), если не отличается от остальных? Собственно, Эйнштейн отказался от светоносного эфира при формулировке специальной теории относительности вскоре после того, как Пуанкаре показал необнаружимость движения относительно эфира.

              Хотя, в статьях о космологии, ученые сплошь и рядом используют именно абсолютные координаты, хоть и явно это не объявляют, потому что заклюют.

              Это вы про сопутствующие координаты?

              Например скорости измеренные относительно реликтового излучения вполне тянут на абсолютные скорости.

              Вроде бы вы тут преувеличиваете. К реликтовому излучению лишь удобно привязаться, потому что оно везде есть. Характерная дипольная анизотропия может возникать не только от относительного движения, но и от очень крупномасштабных возмущений в ранней Вселенной, эти эффекты просто не отделяются друг от друга на практике.


    1. saboteur_kiev
      28.11.2023 17:50
      +1

      «Движение внутренне-присущее свойство материи». И этот ответ меня не удовлетворяет, так как по сути он означает «Мы не знаем. Все движется просто так само собой.»

      А если сказать, что движение присуще такому параметру как время?
      Что такое энергия без времени?
      А что такое время, а еще интереснее как его измерять в разных условиях?


      1. johnfound
        28.11.2023 17:50

        А что такое время, а еще интереснее как его измерять в разных условиях?

        Как его измерять как раз не очень интересно. Если есть понимание сути, то измерить по любому сумеем. ;)


        1. saboteur_kiev
          28.11.2023 17:50
          +1

          Ну вот не всегда. Также как измерить температуру космоса, где количество молекул на метр квадратный несопоставим земной атмосфере. Или что такое температура атома? =)


      1. eton65
        28.11.2023 17:50

        А если сказать, что движение присуще такому параметру как время?

        Скорее пространству. Но эту сущность человечество только-только начинает изучать.


    1. Gorkavyi Автор
      28.11.2023 17:50

      "А если сила отсутствует, то координаты все-равно меняются беспричинно."

      Если что-то движется (меняет координаты), то это означает, что когда-то к этому объекту была применена сила. Например, возьмите набор покоящихся частиц - как только вы отпустите их, они начнут двигаться, сжимаясь в облако под действием тяготения.


      1. johnfound
        28.11.2023 17:50

        то это означает, что когда-то к этому объекту была применена сила.

        Согласен, но тогда выходит, что у материи есть какой-то механизм памяти, который помнит прошлые воздействия. Понимание как работает эта «память» вероятно даст ответ и на мой вопрос из первого поста – о движении.


        1. rombell
          28.11.2023 17:50
          +1

          эта память и есть движение само по себе


          1. MishaRash
            28.11.2023 17:50

            Инерция, можно сказать — что начало двигаться, то и продолжает, пока не появилась замедляющая сила.


            1. rombell
              28.11.2023 17:50

              Именно так. Замедляющая или отклоняющая.


          1. johnfound
            28.11.2023 17:50

            Не думаю. Получается какая-то закольцованная дефиниция, которая опять ничего не объясняет, кроме "все движется, потому что движется".


            1. rombell
              28.11.2023 17:50

              Это потому, что Вы пытаетесь подобрать новый термин для того, для чего термин уже существует. "Память о действовавших силах" - это текущее движение.


            1. codecity
              28.11.2023 17:50

              Получается какая-то закольцованная дефиниция, которая опять ничего не объясняет, кроме "все движется, потому что движется".

              Ну вот вопрос о трех измерениях - вам нужно доказательство что есть только 3 измерения в нашем пространстве - или это аксиома? Время можно рассматривать как четвертое - однако же оно не обладает теми же свойствами, что и другие 3 измерения - мы не можем произвольно перемещаться по временной шкале, невозможно вращение - постоянно только вперед. Или же это особенности нашего восприятия и можно воспринимать как-то иначе?

              Если принять время за 4 измерение пространства - то движение это всего лишь форма объекта в этом измерении. Придали шару движение - просто растянули его как резинку в проекции этого измерения. Точнее даже не так - по времени практически все тела будут растянуты как колбаски (кроме изменяющих форму), а тела в движении будут иметь другую форму - как бы под углом к другим осям.


              1. johnfound
                28.11.2023 17:50

                Ну, чтобы мы не могли воспринимать время как направление, достаточно чтобы электромагнитные волны не распространялись в это направление. И они очевидно туда не распространяются.

                Мы время конечно воспринимаем как-то, но это восприятие неполное, по нему нельзя сказать пространственное ли это измерение или такое ощущение является иллюзией.


                1. codecity
                  28.11.2023 17:50

                  И они очевидно туда не распространяются.

                  С этим не поспоришь.

                  Однако же может быть некая устранимая причина, которая делает 4-е измерение особенным. Причина, которой мы пока не понимаем.

                  Кривая аналогия. Вот, у нас на земле - мы без особых (относительно) усилий движемся по X, Y -осям, а вот по Z-оси - уже так просто не получается. И виной всему - гравитация Земли, очень сложно ее преодолеть. В условиях невесомости - разницы между тремя осями нет.

                  Возможно есть некая подобная причина, которая и ось T делает совсем особенной, хотя в других условиях она бы такой не была.


                  1. edo1h
                    28.11.2023 17:50

                    А закон сохранения? Он тоже не работает, если вместо оси времени выбрать любую другую.

                    Да и три оси в пространстве мы можем выбрать любым способом, например, можем повернуть две оси относительно третьей. При этом координаты пересчитаются, X1 и Y1 будут образованы «смешением» X и Y, но вся физика продолжит работать так же. Такого поворота с участием оси времени я даже представить не могу, вся физика сломается.


                    1. codecity
                      28.11.2023 17:50

                      А закон сохранения? Он тоже не работает, если вместо оси времени выбрать любую другую.

                      Это если учесть гравитацию Земли (в наших условиях). А вот людям, которые о гравитации не знали - не казалось что все три оси симметричны - вверх мяч подбросишь и он возвращается, всегда падает, а вот вперед или влево - такого эффекта нет. Возможно с временной осью та же байда - некая причина, которая делает эту ось особенной именно в наших условиях.


                      1. ksbes
                        28.11.2023 17:50

                        Рудиментарные понятия гравитации были уже тогда по тогда: элемент земли в предмете к земле притягивается. А вот воздух - к земле в такой схеме не притягивается. И свободен двигаться во всех трёх измерениях. Так что аналогия - не очень.

                        d/dх не много где встречается. А вот d/dt - постоянно и на d/dx хрен заменишь. А всё потому что есть фундаментальное интуитивное понятие движения, которое чётко выделят понятие время и выделяет его из других измерений.


        1. Wizard_of_light
          28.11.2023 17:50

          у материи есть какой-то механизм памяти

          Состояния, не изменяющиеся сами по себе, без взаимодействия с другими частями материи. Или, если переходить к представлению Нетёр, симметрии, не нарушающиеся без взаимодействия.


      1. PrinceKorwin
        28.11.2023 17:50

        Например, возьмите набор покоящихся частиц

        Частицы никогда не находятся в покое. Даже при абсолютном нуле они двигаются. Нет никакой возможности создать условия при которых частицы абсолютно покоятся.


    1. eton65
      28.11.2023 17:50

       Почему для изменения первой производной мы видим причину (силу) а для изменения координат нет?

      Вопрос можно задать по другому - относительно чего мы вообще движемся (физически и математически)? Если относительно ткани пространства, то как оно "понимает" что мы движемся, или не движемся - какие шестеренки мы при этом задеваем?


      1. Gorkavyi Автор
        28.11.2023 17:50
        +1

        Мы можем двигаться только относительно друг друга и материальных объектов, которые погружены в пространство. Если вы одни в космосе и не ускоряетесь, то ваше движение ничем от покоя не будет отличаться.


        1. eton65
          28.11.2023 17:50
          -1

          Если вы одни в космосе и не ускоряетесь, то ваше движение ничем от покоя не будет отличаться

          А если я один в космосе, то откуда я знаю - ускоряюсь я или лечу с постоянной скоростью?


          1. MishaRash
            28.11.2023 17:50
            +4

            От ускорения вы почувствуете силу инерции. Хотя согласно общей теории относительности можно подумать, что это на самом деле гравитация какого-то вещества (невидимого). Разницу можно понять, если вы начнёте градиенты измерять.


          1. Gorkavyi Автор
            28.11.2023 17:50

            Да, ускорение вас вдавит в кресло, хотя понять - не действует ли тут гравитация планеты - для постоянного ускорения нельзя.


            1. eton65
              28.11.2023 17:50

              хотя понять - не действует ли тут гравитация планеты - для постоянного ускорения нельзя

              Можно - включив и выключив ракетный двигатель.

              Ну и гравитация не вдавит меня в кресло (если только мой корабль не сопротивляется ей).

              Да, ускорение вас вдавит в кресло

              Значит я знаю, что я ускоряюсь. И никакие другие объекты, чтобы понять это, не нужны. А выключив двигатель, я буду продолжать лететь с постоянной скоростью и буду знать об этом. Получается движение абсолютно)


              1. Wizard_of_light
                28.11.2023 17:50
                +1

                А выключив двигатель, я буду продолжать лететь с постоянной скоростью и буду знать об этом.

                Или будете думать что знаете. Может двигатель включает аналог Лоренцевой силы и разворачивает корабль, а скорость всегда одна и та же.


                1. eton65
                  28.11.2023 17:50

                  Может двигатель включает аналог Лоренцевой силы

                  Принцип работы двигателя известен даже школьнику, зачем тянуть сюда фэнтези?


                  1. Zenitchik
                    28.11.2023 17:50

                    Причём тут фэнтези? Двигатель нельзя использовать по нормали к скорости?


        1. rombell
          28.11.2023 17:50
          +1

          С появлением выделенного центра координат (центра Вселенной) уже отличается.

          А "один в космосе" - это про другую Вселенную, законы которой не факт, что применимы к нашей - даже если на первый взгляд похожи


  1. Tzimie
    28.11.2023 17:50
    +5

    Да, Вселенная имеет центр, расположенный, видимо, в направлении южного полюса.

    Там же черепаха!


  1. Tzimie
    28.11.2023 17:50
    +4

    А серьезно, нужно summary на несколько абзацев теории. Особенно интересует поведение энтропии при отскоке


    1. Gorkavyi Автор
      28.11.2023 17:50

      Я наверху ответил про энтропию, даже с картинкой.


    1. Gorkavyi Автор
      28.11.2023 17:50

      Добавлена ссылка на мою свежую статью по энтропии Вселенной (см. добавленный P.S.)


  1. Yukr
    28.11.2023 17:50

    Теория пульсирующей Вселенной выглядит ИМХО интереснее и логичнее Большого взрыва.

    И всё-таки не снимает вопроса - а откуда взялась Мегадыра? Понимаю, что тема книги не в этом, но страшно хочется разобраться.

    Вот цитата из книги автора:

    Как возникла наша замкнутая Вселенная или Мегадыра с ее содержимым? Практически единственный разумный ответ, который возможен при нынешнем понимании космологии: Вселенная возникла вследствие огромной флуктуации. Если существовала бесконечная самогравитирующая среда с небольшой плотностью гравитационных волн и какой-то обычной материи (фотонов, барионов, электронов, нейтрино и т.д.), то всегда можно выделить объем, гравитирующая масса в котором будет достаточна для замыкания пространства или образования огромной черной дыры.

    существовала среда или нет - откуда она взялась? всё остальное, получается, вторично: едет паровоз прямо по бесконечным рельсам, или по замкнутому кольцу - не так интересно, как откуда он взялся??

    Есть идеи?

    (чисто чтобы сэкономить место и усилия осведомлённых комментаторов, скажу, что притчу о вопросе Малункьяпутты к Будде я знаю)))


    1. Gorkavyi Автор
      28.11.2023 17:50
      +1

      Есть приоритеты задач. Когда вы смотрите на часы, механизм действия которых вы должны понять, то вас в первую очередь интересует существующее положение вещей (шестеренок и пружин). Я был сфокусирован на этой задаче. Второй вопрос - кто (или что) создал эти часы и из каких материалов? Это другой вопрос, на который пока нет определенного ответа, впрочем, на него нельзя будет ответить без понимания механизма действия часов.


      1. Yukr
        28.11.2023 17:50

        Согласен. Удачи!


  1. ussrisback
    28.11.2023 17:50

    Если Вселенная расширяется, то какие силы заставят ее начать сжиматься, когда исчезнет последний атом?


    1. Gorkavyi Автор
      28.11.2023 17:50
      +1

      Она расширяется по инерции, а в ее центре уже растет Большая Черная Дыра, которая нас догонит, развернет, проглотит и заставит двигаться к ее центру.


      1. commanderkid
        28.11.2023 17:50

        Правильно ли я понял, что ЧД не разрушаются в новом цикле и остаются со старого, передаются так сказать по наследству?


        1. Gorkavyi Автор
          28.11.2023 17:50
          +2

          Да, черные дыры неразрушимы по определению (излучением Хокинга пренебрегается), они могут только образовываться, расти и сливаться. Стационарность этого постоянно растущего населения, передаваемого из цикла в цикл, достигается тем, что в каждом цикле из популяции убирается самая Большая Дыра - мы в нее проваливаемся, избавляясь как от нее, так и от основной массы энтропии, которая содержится в ее поверхности.


          1. PrinceKorwin
            28.11.2023 17:50

            > Да, черные дыры неразрушимы по определению (излучением Хокинга пренебрегается), они могут только образовываться, расти и сливаться

            Но ведь при слиянии черных дыр результирующая ЧД имеет массу меньше их сумм. Т.е. часть массы ЧД именно что разрушена - переведена в энергию гравитационных волн. И это уже наблюдаемый/зарегистрированный процесс.


            1. Gorkavyi Автор
              28.11.2023 17:50

              Речь идет о том, что каждая отдельная ЧД не может уменьшать свою массу. Две дыры, сливаясь, уменьшают свою суммарную массу, но это не нарушает данное утверждение.


              1. PrinceKorwin
                28.11.2023 17:50

                Ну нет же. Эта энергия на гравитационные волны была взята именно из черной дыры. Точнее из обоих. Т.е. существует доказанный процесс уменьшения массы ЧД. Для этого даже слияния не нужно. Достаточно чтобы две ЧД имели общий центр масс вокруг которого будут кружить.


                1. Gorkavyi Автор
                  28.11.2023 17:50

                  Будьте внимательны: ОТДЕЛЬНАЯ черная дыра не может уменьшать свою массу. Две дыры, сливаясь уменьшают свою суммарную массу, но и исчезают при этом как отдельные дыры. А суммарная дыра - снова отдельная и снова не может уменьшать свою массу. Когда две дыры кружатся друг вокруг друга, они сбрасывают в гравволны не свою массу, а энергию орбитального движения.


                  1. PrinceKorwin
                    28.11.2023 17:50

                    Отдельная ЧД может испаряться за счёт излучения Хокинга. Правда этот эффект тем сильнее, чем меньше масса ЧД. Для БЧД он будет уже принебрежимо мал. Но тем не менее такой эффект есть.

                    Также ЧД вращаются. Просто по причине получения вращательного момента от падающей на них материи. И также доказано уже, что это вращение закручивает вокруг ЧД пространство-время в радиусе большем чем сфера Шварцшильда. На что тратится энергия.

                    гравволны не свою массу, а энергию орбитального движения

                    Нет. В системе вращающийся ЧД вокруг общего центра масс именно масса обеих ЧД тратится на создание гравитационных волн. Почему? Поэтому что если бы тратилась энергия орбитального движения (как вы это называете), то скорость вращения вокруг центра масс уменьшалась. Но в реальности она только увеличивается.


                    1. Gorkavyi Автор
                      28.11.2023 17:50

                      "Отдельная ЧД может испаряться за счёт излучения Хокинга. Правда этот эффект тем сильнее, чем меньше масса ЧД. Для БЧД он будет уже принебрежимо мал. Но тем не менее такой эффект есть."

                      В реальности его нет, потому что температура реликтового эм излучения всегда больше температуры БЧД, поэтому этот эффект забивает любое квантовое испарение.

                      Извините, но прежде чем браться за исследования черных дыр, вам надо исследовать закон Кеплера. При уменьшении орбитальной энергии, орбита сжимается, а скорость вращения УВЕЛИЧИВАЕТСЯ - по формулам средневековья.


                      1. commanderkid
                        28.11.2023 17:50

                        Ок. "Вселенная" которая провалилась в новую черную дыру будет иметь массу меньше, чем "предыдущая" вселенная?


                      1. Gorkavyi Автор
                        28.11.2023 17:50

                        Это из-за чего такой урон? Сколько вещества породило МегаДыру, столько войдет и в Большую Дыру. Энергия полностью сохраняется внутри дыры.


          1. freewind
            28.11.2023 17:50

            То есть получается не циклическая вселенная, а мы с каждым "большим взрывом" погружаемся всё глубже и глубже в центральную большую чёрную дыру?


            1. Gorkavyi Автор
              28.11.2023 17:50

              Я не знаю, как у вас получился этот удивительный вывод. Вселенная пульсирует в стационарной дыре, то расширяясь из-за антигравитации, то сжимаясь из-за гравитации Большой Дыры. Куда это мы проваливаемся?


      1. PrinceKorwin
        28.11.2023 17:50
        +1

        Погодите. Гравитационное взаимодействие распространяется со скоростью света. Но вот расширение пространства этой скоростью не ограничено и это явление наблюдаемо.

        Вопрос: как эта мега черная дыра сможет догнать остальную материю во вселенной если двигается медленнее ее удалению?


        1. Gorkavyi Автор
          28.11.2023 17:50

          А что мешает поверхности Большой Черной Дыры расширяться со сверхсветовой скоростью? Ведь это не материальная, а воображаемая (вычисляемая) граница.


          1. PrinceKorwin
            28.11.2023 17:50

            Потому, что эта воображаемая граница является следствием массы ЧД. Гравитационный радиус пропорционален массе тела M и равен r_g = 2TGM/c^2

            Т.е. растет медленнее чем поедает массу. Поэтому он не может расти быстрее скорости света.


            1. Gorkavyi Автор
              28.11.2023 17:50

              На странице 315 моей книжки рассматривается этот вопрос. Из приведенных вами формул никак не следует, что приток массы как-то ограничивается, что может ограничивать скорость расширения ЧД. Напомню, что БЧД питается в основном гравитационными волнами, которые сами налетают на нее со скоростью света.


              1. PrinceKorwin
                28.11.2023 17:50

                Не следует, но вы можете посчитать сколько нужно поглащать массы ЧД чтобы сфера Шварцшильда росла со скоростью света. При этом нужно помнить, что нужно поглотить эту массу не единоразово, а в течение длительного времени.

                Напомню, что БЧД питается в основном гравитационными волнами, которые сами налетают на нее со скоростью света.

                Думаю вы также посмотрите, что гравитационная сила - это самая слабая из возможных сил которую мы знаем. И она реально очень малая. Если вы так уверенно говорите, что этой силы хватит ЧД чтобы расти со световой скоростью, то покажите, пожалуйста расчеты.

                Если же этого не достаточно. То опять мой вопрос. Как же эта ЧД сможет догнать ту часть вселенной которая отдаляется от нее со скоростью, превышающую скорость света?


                1. Gorkavyi Автор
                  28.11.2023 17:50
                  +2

                  На стр. 319-320 моей книги такие расчеты проделаны: "Логично предположить, что когда радиус Большой Черной Дыры будет сопоставим с радиусом Вселенной, то радиус этой дыры, из-за поглощения фонового гравитационного излучения, будет увеличиваться со скоростью, близкой к скорости света. Из (22) можно записать условие такого роста:"

                  Но можно зайти и с другой стороны: Вселенная расширяется, но находится в черной дыре (МегаДыре), следовательно, рано и ли поздно ее расширение остановится и все вещество начнет сжиматься. Растущая Большая Черная Дыра, как бы медленно она не росла, все равно поглотит все сжимающееся вещество Вселенной и станет равна МегаДыре. То есть, по сути неважно, что развернет расширяющееся вещество - внутренняя граница стационарной МегаДыры или внешняя граница растущей Большой Черной Дыры. Вот такая устойчивость сценария - верный признак его верности. Об этом говорит весь мой сорокакалетний опыт физика-теоретика.


                  1. PrinceKorwin
                    28.11.2023 17:50

                    радиус этой дыры, из-за поглощения фонового гравитационного излучения, будет увеличиваться со скоростью, близкой к скорости света.

                    Т.е. если БЧД будет даже размером со вселенную, то это ей не поможет догнать ту часть материи которая отдаляется со скоростью более света. Оок.

                    Вселенная расширяется, но находится в черной дыре (МегаДыре), следовательно, рано и ли поздно ее расширение остановится и все вещество начнет сжиматься.

                    Вообще не "следовательно". С чего это эта Мега БЧД должна в какой-то момент начать сжиматься? Потому, что не будет материала для поглощения? В этом случае она просто перестанет расти. Почему будет сжатие? Мы уже наблюдаем много ЧД и пока нет ни одного наблюдения показывающее, что они сжимаются.

                    Более того, вы сами же выше и писали, что нет никаких процессов для уменьшения ЧД (я там спросил про систему из двух ЧД вращающихся вокруг общего центра масс, кстати).


                    1. Gorkavyi Автор
                      28.11.2023 17:50

                      Вы что-то крупно не поняли. Не черные дыры будут сжиматься, а вещество внутри них - для внутреннего наблюдателя, который летит с этим веществом. Именно это вещество и составляет наблюдаемую часть Вселенной.


                      1. PrinceKorwin
                        28.11.2023 17:50

                        Процитирую вас ещё раз:

                        следовательно, рано и ли поздно ее расширение остановится и все вещество начнет сжиматься.

                        Тогда, по вашим словам выходит, что вещество внутри ЧД сначала разлетается, а потом сжимается? Как так?


                      1. Gorkavyi Автор
                        28.11.2023 17:50

                        Именно так. Сжатие вещества внутри черной дыры происходит вполне по классическим схемам гравитационного коллапса, но когда на финальной стадии коллапса (большая часть вещества Вселенной занимает объем с размером около 10 светолет) черные дыры начинают активно сливаться, генерируя гравитационные волны, возникает сильная антигравитация, преодолевающая притяжение (и ликвидирующая любые сингулярности) и снова заставляющая вещество расширяться - это и есть Большой Взрыв. Когда частота слияния падает, то динамика Вселенной снова становится более-менее классической (с поправкой из-за роста БЧД).


                      1. PrinceKorwin
                        28.11.2023 17:50

                        генерируя гравитационные волны, возникает сильная антигравитация

                        Разве грав.волны отталкивают материю? Откуда появится антигравитация?


                      1. Gorkavyi Автор
                        28.11.2023 17:50

                        Гравитационные волны ничего не отталкивают. То есть вы категорически не хотите прочитать ни популярные статьи в "Науке и Жизни", ни популярную книгу на странице 130 (см. рис), а хотите, чтобы я вам все пересказал кратенько? Мило.


                      1. PrinceKorwin
                        28.11.2023 17:50

                        В вашей теории мне не понятен следующий момент. Если мы находимся в БЧД, то должны быть видимы следующие эффекты:

                        • ЧБД вращается (получая момент от поглашаемого вещества)

                        • Т.к. процесс поглощения/роста БЧД не константа (поглощаемое вещество не равномерно в пространстве по плотности), то мы бы это наблюдали на макроуровне как рябь. Но мы этого не наблюдаем.

                        • В ситуации когда наша ЧБД встречается с подобной себе происходило бы сначала большой сброс массы на внешние нрав.волны, а потом очень резкий скачок роста/погашения массы. Это бы мы точно увидели, но ничего подобного не наблюдается.

                        • Если бы такой процесс был, то из-за центробежных сил был виден нами не как пузырь, но как артефакт протяженностью на всю вселенную

                        Следовательно если мы находимся в БЧД и видим то, что видим это может быть только при:

                        • Материя вне БЧД равномерная по плотности на всем протяжении пространства - время

                        • Наша ЧБД единственная и ни с кем не пересекается/поглощается

                        И да. Т.к. при погашении внешней материи растет масса ЧБД, то чтобы сохранять скорость роста внешняя материя должна равномерно вокруг БЧД увеличить свою плотность строго пропорционально росту массы БЧД. Иначе скорость роста падает чего мы тоже не наблюдаем.


                      1. Gorkavyi Автор
                        28.11.2023 17:50

                        Ваши "должны быть" и "тоже ненаблюдаем" следуют только из ваших каких-то внутренних умопостроений и убеждений (в которые пока понимание законов Кеплера не входит). Но я не могу вам что-то ответить, пока вы не разберетесь в разнице между МегаДырой и Большой Черной Дырой - см. хотя бы рисунок из книжки:


                      1. PrinceKorwin
                        28.11.2023 17:50

                        Ваши "должны быть" и "тоже ненаблюдаем" следуют только из ваших каких-то внутренних умопостроений и убеждений

                        Если вам не сложно, то приведите, пожалуйста, ссылки на эти наблюдаемые явления. Раз "не наблюдаем" вы не принимаете.

                        И скатывание обсуждения с вашей территории к моей личности никак не поднимает уровень доверия к вашей территории.

                        Но я не могу вам что-то ответить, пока вы не разберетесь в разнице между МегаДырой и Большой Черной Дырой

                        Эта картинка из вашей книжки подкреплена наблюдениями? Можно ссылки не только на материал из вашей книги, но и на других авторов?


                      1. rombell
                        28.11.2023 17:50

                        Вы обсуждаете представленную теорию. Требовать ссылки на других авторов просто глупость, извините.

                        Отвечу за автора:

                        >ЧБД вращается (получая момент от поглашаемого вещества)

                        БЧД внутри не монолит, а совокупность объектов. Поглощёное вещество сохраняет свой импульс. Нет механизма, по которому поглощённое на окраине вещество передаст импульс внутренним слоям, пока не долетит.

                        >Т.к. процесс поглощения/роста БЧД не константа (поглощаемое вещество не равномерно в пространстве по плотности), то мы бы это наблюдали на макроуровне как рябь. Но мы этого не наблюдаем.

                        Вы постулируете, что мы должны наблюдать рябь. Рябь чего? Почему мы должны её наблюдать? Каков механизм её возникновения?

                        >В ситуации когда наша ЧБД встречается с подобной себе 

                        Она одна такая by def

                        >Если бы такой процесс был, то из-за центробежных сил был виден нами не как пузырь, но как артефакт протяженностью на всю вселенную

                        Пузырь чего? Вы не можете заглянуть внутрь другой ЧД. Если она поглощена нашей БЧД, для нас она по-прежнему будет видна как ЧЖ независимо от её размеров и видимости снаружи нашей БЧД


  1. a999x94
    28.11.2023 17:50
    -5

    Ну вы почти у цели! )

    Тут вы найдёте подтверждение и много интересного. Кто эта женщина даже не пытайтесь выяснить) она сейчас ушла из поля зрения. Я был лично с ней знаком и то что она могла делать это за гранью нашей физики

    https://youtu.be/GDxyEmnANok?t=231


  1. alartus
    28.11.2023 17:50
    -5

    Сначала подумал что статья про глистов, заголовок обманчивый.


  1. anatolykern
    28.11.2023 17:50

    Мне больше всего понравилось предсказание о том, что цикл в 100 млрд лет идёт с пульсацией вселенной в размере от ~10 до ~1 трлн св лет, т.е. мы на подростковом этапе развития.


  1. bigfoot_tmn
    28.11.2023 17:50
    +2

    Уважаемые читатели, продам вечные иглы для примуса, недорого!


  1. Gorkavyi Автор
    28.11.2023 17:50

    Статья расширена на P.S. со ссылкой на мою свежую статью об энтропии циклической Вселенной. Делаем науку прямо на глазах!


  1. anatolykern
    28.11.2023 17:50

    По поводу вопроса, может малость наивного, как от далекого от космологии:

    Можно ли определить центр масс и геометрию аккреционного диска BBH?

    Есть ли факторы возможного неравномерного расширения пространства и неравномерности скоростей вращения вокруг центра масс, влияющие на положение солнечной системы по отношению к векторному представлению от этого центра?